MCAT General Chemistry Discrete Study Questions

¡Supera tus tareas y exámenes ahora con Quizwiz!

(12-12) For a cell with the following half-reactions: anode: SO2 + 2 H2O -> SO4^2- + 4 H+ + 2e- cathode: Pd2+ + 2e- -> Pd How would decreasing the pH of the solution inside the cell affect the electromotive force (emf)? A) the emf would decrease B) the emf would remaind the same C) the emd would increase D) the emf would become 0

A A change in pH has a direct correlation to the hydrogen ion (H+) concentration. Decreasing the pH increases the H+ concentration, which means the concentration of products has increased in the oxidation of sulfur dioxide. This means it would be harder to liberate electrons, thus decreasing the emf. One could also view this decrease in oxidation potential as an increase in reduction potential. If E*red,anode decreased, then E*cell must decrease.

(6-15) Which of the following statements best describes the effect of lowering the temperature of the following reaction? A + B <--> C + D deltaH= -1.12 kJ/mol A) [C] and [D] would increase B) [A] and [B] would increase C) deltaH would increase D) deltaH would decrease

A A negative deltaH value indicates an exothermic reaction, meaning that the forward reaction produces heat. Visualize this as: A + B <--> C + D + heat This means that removing heat by decreasing the temperature is similar to removing any other product of the reaction. To compensate for this loss, the reaction will shift to the right, causing an increase in the concentrations of C and D as well as a decrease in the concentrations of A and B.

(7-10) A chemical reaction has a negative enthalpy and a negative entropy. Which of the following terms necessarily describes this reaction? A) exothermic B) endothermic C) exergonic D) endergonic

A A reaction with a negative enthalpy is, by definition, exothermic. Because both enthalpy and entropy are negative, this is a temperature-dependent process, and the reaction will be both endergonic and exergonic- but only at particular temperatures, eliminating C and D.

(10-3) Which of the following represents chloric acid? A) HClO3 B) ClO3^- C) HClO2 D) HClO

A Acids ending in -ic are derivatives of anions ending in -ate, while acids ending in -ous are derivatives of anions ending in -ite. CLO3^- is chlorate because it has more oxygen than the other commonly occurring ion, CLO2^-, which is named chlorite. Therefore, HClO3 is chloric acid. HClO2 represents chlorous acid. HClO represents hypochlorous acid.

(2-2) Carbon and silicon are the basis of biological life and synthetic computing, respectively. While these elements share many chemical properties, which of the following best describes a difference between the two elements? A) carbon has a smaller atomic radius than silicon B) silicon has a smaller atomic radius than carbon C) carbon has fewer valence electrons than silicon D) silicon has fewer valence electrons than carbon

A As one moves from top to bottom in a group (column), extra electron shells accumulate, despite the fact that the valence configurations remain identical. These extra electron shells provide shielding between the positive nucleus and the outermost electrons, decreasing the electrostatic attraction and increasing the atomic radium. Because carbon and silicon are in the same group, and silicon is farther down in the group, silicon will have a larger atomic radius because of its extra electron shell.

(7-5) Methanol reacts with acetic acid to form methyl acetate and water. (**insert pic here**) Based on the values in the table, what is the heat of reaction in kJ/mol? A) 0 B) 464 C) 824 D) 1288

A At first glance, this might seem like a math-heavy problem, but it really doesn't require any calculations at all. We just have to keep track of which bonds are broken and which bonds are formed. Remember, breaking bonds required energy, while forming bonds releases energy. Two bonds are broken: a C-O bond between the carbonyl carbon and oxygen of acetic acid and an O-H bond between the hydroxyl oxygen and hydrogen of methanol. Two bonds are also formed: a C-O bond between a hydroxyl group and a hydrogen to form water. Given that the same two bonds are broken and formed in this reaction, the energy change is 0 kJ/mol.

(5-4) The concentrations of all reactants in a zero-order reaction are increased twofold. What is the new rate of the reaction? A) it is unchanged B) it is decreased by a factor of 2 C) it is increased by a factor of 2 D) it cannot be determined from the information given

A By definition, zero-order reactions are unaffected by the concentrations of any reactants in the reaction. Thus, changing the concentrations of these reactants will not affect the rate.

(6-3) Carbonated beverages are produced by dissolving carbon dioxide in water to produce carbonic acid: CO2 (g) + H2O (l) <--> H2CO3 (aq) When a bottle containing carbonated water is opened, the taste of the beverage gradually changes as the carbonation is lost. Which of the following statements best explains this phenomenon? A) The change in pressure and volume causes the reaction to shift to the left, thereby decreasing the amount of aqueous carbonic acid B) the change in pressure and volume causes the reaction to shift to the right, thereby decreasing the amount of gaseous carbon dioxide C) carbonic acid reacts with environmental oxygen and nitrogen D) carbon dioxide reacts with environmental oxygen and nitrogen

A Carbon dioxide gas evolves and leaves the bottle, which decreases the total pressure of the reactants. Le Chatelier's principle explains that a decrease in pressure shifts the equilibrium to increase the number of moles of gas present. this particular reaction will shift to the left, which in turn will decrease the amount of carbonic acid and increase the amount of carbon dioxide and water. Oxygen and nitrogen are not highly reactive and are unlikely to combine spontaneously with carbon dioxide or carbonic acid (C and D).

(7-13) A reaction coordinate for a chemical reaction is displayed in the graph below (**insert pic here**) Which of the following terms describes the energy of this reaction? A) endothermic B) exothermic C) endergonic D) exergonic

A Eliminate C and D, which describe the free energy of reaction and cannot be determined from this graph. While most reaction coordinate graphs we've explored in this book use free energy for the y-axis, this one uses potential energy (enthalpy). If the heat of formation of the products is greater than that of the reactants, the reaction is endothermic. We can determine this information from their relative positions on the graph: because the products are higher than the reactants, this is an endothermic reaction.

(4-3) Which of the following is the gram equivalent weight of H2SO4 with respect to protons? A) 49.1 g B) 98.1 g C) 147.1 g D) 196.2 g

A First, it is helpful to know the molar mass of one mole of H2SO4, which is found by adding the atomic weights of the atoms that constitute the molecule (98.1 g/mol H2SO4). Gram equivalent weight is the mass in grams that would release one mole of protons. Because sulfuric acid has two hydrogens per molecule, the gram equivalent weight is 98.1 g divided by 2 (49.1 g).

(8-1) Which of the following sets of conditions would be least likely to result in ideal gas behavior? A) high pressure and low temperature B) low temperature and large volume C) high pressure and large volume D) low pressure and high temperature

A Gases deviate from ideal behavior at higher pressures and lower volumes and temperatures, all of which force molecules closer together. The closer they are, the more they can participate in intermolecular forces, which violates the definition of an ideal gas. At low temperatures, the kinetic energy of the particles is reduced, so collisions with other particles or the walls of the container are more likely to result in significant changes in kinetic energy.

(11-3) Lithium aluminum hybride (LiAlH4) is often used in labs because of its tendency to donate a hybride ion. Which of the following roles would lithium aluminum hybride likely play in a reaction? A) strong reducing agent only B) strong oxidizing agent only C) both a strong reducing agent and strong oxidizing agent D) neither a strong reducing agent nor a strong oxidizing agent

A Hydride ions are composed of a hydrogen nucleus with two electrons, thereby giving it a negative charge and a considerable tendency to donate electrons. LiAlH4 is therefore a strong reducing agent. Strong reducing agents tend to have metals or hydrides; strong oxidizing agents tend to have oxygen or a similarly electronegative element.

(5-7) A certain chemical reaction has the following rate law: rate=k[NO2][Br2] Which of the following statements necessarily describes the kinetics of this reaction? I. the reaction is second-order II. the amount of NO2 consumed is equal to the amount of Br2 consumed III. the rate will not be affected by the addition of a compound other than NO2 and Br2 A) I only B) I and II only C) II and III only D) I, II, and III

A If the sum of the exponents (orders) of the concentrations of each species in the rate law is equal to 2, then the reaction is second-order. The exponents in the rate law are unrelated to stoichiometric coefficients, so NO2 and Br2 could have any stoichiometric coefficients in the original reaction and still be a second-order reaction, invalidating statement II. Statement III is incorrect because the rate can be affected by a wide variety of compounds. A catalyst, for example, could increase the rate.

(9-9) Which of the following combinations of liquids would be expected to have a vapor pressure higher than the vapor pressure that would be predicted by Raoult's law? A) ethanol and hexane B) acetone and water C) isopropanol and methanol D) nitric acid and water

A Mixtures have a higher vapor pressure than predicted by Raoult's law and have stronger solvent-solvent and solute-solute interactions than solvent-solute interactions. Therefore, particles do not want to stay in solution and more readily evaporate, creating a higher vapor pressure than an ideal solution. Two liquids that have different properties, like hexane (hydrophobic) and ethanol (hydrophilic, small) (A) would not have many interactions with each other and would cause positive deviation; higher vapor pressure. B and C are composed of liquids that are similar to one another and would not show significant deviation from Raoult's law. D contains two liquids that would interact very well with each other, which would actually cause a negative deviation from Raoult's law- when attracted to one another, solutes and solvents prefer to stay in liquid form and have lower vapor pressure than predicted by Raoult's law.

(4-2) Which of the following compounds has a formula weight between 74 and 75 grams per mole? A) KCl B) C4H10O C) MgCl2 D) BF3

A Of the compounds listed, both B and D are covalent compounds and thus are measured in molecular weights, not formula weights. the formula weight of MgCl2 is much too high (24.3 amu +2x35.5 amu= 95.3 amu per formula unit), eliminating C. Only KCl fits the criteria (39.1 amu + 35.5 amu = 74.5 amu).

(6-4) What is the proper equilibrium expression for the following reaction? 2 NO2 (g) + 4 H2 (g) <--> N2 (g) + 4 H2) (g) (**insert pic here**)

A Recall that equilibrium constants are either based on concentrations (Kc) or partial pressures (Kp). In this case, because all species are in the gas phase, we are using Kp- eliminating B and D. When water is in the liquid phase, it does not appear in equilibrium expressions, as in C. Here, however, water is in the gaseous phase and thus should appear in the equilibrium expression.

(1-8) Which of the following isotopes of carbon is least likely to be found in nature? A) 6C B) 12C C) 13C D) 14C

A Recall that the superscript refers to the mass number of an atom, which is equal to the number of protons plus the number of neutrons present in an element. Sometimes a text will list the atomic number, Z, as a subscript under the mass number, A. According to the periodic table, carbon contains 6 protons; therefore, its atomic number is 6. Isotopes all have the same number of protons, but differ in the number of neutrons. Almost all atoms with Z>1 have at least one neutron. Carbon is most likely to have a mass number of 12, for 6 protons and 6 neutrons, as in choice B. C and D are possible isotopes that would have more neutrons than 12C. The 6C isotope is unlikely. It would mean that there are 6 protons and 0 neutrons. This would be a highly unstable isotope.

(11-12) Consider the following steps in the reaction between oxalic acid and chlorine: I. Cl2 + H2O --> HOCl + Cl- + H+ II. H2C2O4 --> H+ + HC2O4^- III. HOCl + HC2O4^- --> H2O + Cl- + 2 CO2 Which of these steps, occurring in aqueous solution, is an example of a disproportionation reaction? A) I only B) III only C) I and II only D) I, II, and III

A Step I is a disproportionation reaction because chlorine starts with an oxidation state of 0 in the reactants and ends up with an oxidation state of +1 in HOCl and -1 as Cl-. In the other reactions, no element appears with different oxidation states in two different products. Therefore, only step I is a disproportionation reaction.

(6-10) In a sealed 1L container, 1 mole of nitrogen gas reacts with 3 moles of hydrogen gas to form 0.05 moles of NH3 at equilibrium. Which of the following is closest to the Kc of the reaction? A) 0.0001 B) 0.001 C) 0.01 D) 0.1

A The balanced equation is: N2 + 3 H2 <--> 2 NH3 Kc = Keq = [NH3]^2 / [N2] [H2]^3 Because the volume is 1L, the amount of each gas in moles is equal to the value of the concentration of each gas in moles per liter (M). The relatively small amount of NH3 produced indicates that it will be possible to consider the amount of N2 and H2 that reacts to be negligible compared to their starting concentrations. Putting the amounts of each reactant and product into the Keq expression gives: Keq = [NH3]^2 / [N2] [H2]^3 = [.05]^2 / [1-x] [3-x]^3 ~ [0.05]^2 / [1] [3]^3 = [5x10^-2]^2 /27 = 25x10^-4 /25 = 1x10^-4 = 0.0001.

(3-6) Which of the following has the largest dipole moment? A) HCN B) H2O C) CCl4 D) SO2

A The best way to approach this problem is to draw the structure of each of these molecules, then consider the electronegativity of each bond as it might contribute to an overall dipole moment. HCN is correct because of the large differences in electronegativity aligned in a linear fashion. There is a strong dipole moment in the direction of nitrogen, without any other moments cancelling it out. Water has two dipole moments, one from each hydrogen pointing in the direction of the oxygen. The molecule is bent, and the dipole moments partially cancel out. There is a molecular dipole, but it is not as strong as in HCN. Sulfur dioxide (C) has a similar bent configuration, and its dipole will again be smaller than that of HCN. Further, oxygen and sulfur do not have as large a difference in electronegativity, so even the individual bond dipoles are smaller than those in other molecules. CCl4 (D) has tetrahedral geometry. Although each of the individual C-Cl bonds is highly polar, the orientation of these bonds causes the dipoles to cancel each other out fully, yielding no overall dipole moment.

(6-5) If Kc >> 1: A) the equilibrium mixture will favor products over reactants B) the equilibrium mixture will favor reactants over products C) the equilibrium concentrations of reactants and products are equal D) the reaction is essentially irreversible

A The larger value of Keq (whether Kc or Kp), the larger the ratio of products to reactants. Therefore, if Kc >> 1, there are significantly larger concentrations of products than reactants at equilibrium. Even with a large Keq, the reaction will ultimately reach equilibrium far toward the products side and is therefore reversible, eliminating D.

(4-5) In which of the following compounds is the percent composition of carbon by mass closest to 62%? A) acetone B) ethanol C) propane D) methanol

A The percent composition by mass of any element within a molecule is equal to the mass of that element in the molecule divided by the molar mass of the compound times 100%. In this case, C3H6O~ 66.7%. The actual value would be a little lower, but it is closest to 62% of the choices. B~50%, C~80%, and D~ 37.5%.

(1-12) How many total electrons are in a 133Cs cation? A) 54 B) 55 C) 78 D) 132

A The quickest way to solve this problem is to use the periodic table to find out how many protons are in Cs atoms; there are 55. Neutral Cs atoms would also have 55 electrons. A stable Cs cation will have a single positive charge because it has one unpaired s-electron. This translates to one fewer electron than the number of protons or 54 electrons.

(8-9) At sea level and 25*C, the solubility of oxygen gas in water in 1.25x10^-3 M. In Denver, a city that lies high above sea level, the atmospheric pressure is 0.800 atm. What is the solubility of oxygen in water in Denver? A) 1.00x10^-3M B) 1.05x10^-3M C) 1.50x10^-3M D) 2.56x10^-3M

A The solubility of gases in liquids is directly proportional to the atmospheric pressure, as shown by Henry's law. (1.25x10^-3M)/(1.000 atm) = [O2]2/(0.8000 atm) [O2]2= (0.8x1.25x10^-3)/1= 8/10 x 5/4 x 10^-3= 1x10^-3M

(7-9) Suppose deltaG*rxn= -2000 kJ/mol for a chemical reaction. At 300K, what is the change in Gibbs free energy in kJ/mol? A) deltaG= -2000 + (300K)(8.314)(lnQ) B) deltaG= -2000 - (300K)(8.314)(lnQ) C) deltaG= -2000 + (300K)(8.314)(logQ) D) deltaG= -2000 - (300K)(8.314)(logQ)

A This problem asks for the free energy of a reaction at nonstandard conditions, which can be determined with the equation deltaG= deltaG* + RTlnQ.

(7-1) Consider the cooling of an ideal gas in a closed system. This process is illustrated in the pressure-volume graph shown in the figure (** insert pic here**) Based on this information, the process may be: A) adiabatic B) isobaric C) isothermal D) isochoric

A This process may be adiabatic. Given that the gas was cooled, it did not maintain constant temperature, eliminating C. Isobaric and isovolumetric processes appear as horizontal and vertical lines in pressure-volume graphs, respectively, eliminating B and D. Adiabatic processes appear hyperbolic on pressure-volume graphs, as illustrated.

(4-7) In the reaction shown, if 39.05 g of Na2S are reacted with 85.5 g of AgNO3, how much of the excess reagent will be left over once the reaction has gone to completion? Na2S + 2 AgNO3 --> Ag2S + 2 NaNO3 A) 19.5 g Na2S B) 26.0 g Na2S C) 41.4 g AgNO3 D) 74.3 g AgNO3

A We must first determine which species is the limiting reagent. The formula weight of Na2S is 78.1 g/mol Na2S. The formula weight of AgNO3 is 169.9 g/mol AgNO3. From this, it can be determined that we are given: 39.05g Na2S/ 78.1 g/mol = 0.5 mol Na2S and 85.5 g AgNO3/ 169.9 g/mol= 0.5 mol AgNO3. Because we need 2 moles of AgNO3 for every mol of Na2S, AgNO3 is the limiting reagent. The correct answer will be in grams of Na2S. If .5mol AgNO3 are used, Na2S is consumed at half the rate so 0.25 mol Na2S will be used. We then have 0.25 mol excess Na2S, which has a mass of about 20g.

(1-11) Suppose that an atom fills its orbitals as shown: [^v] (3s) [^][^][^] (3p) Such an electron configuration most clearly illustrates which of the following laws of atomic physics? A) Hund's rule B) Heisenberg uncertainty principle C) Bohr model D) Rutherford model

A The MCAT covers the topics in this chapter qualitatively more often than quantitatively. It is critical to be able to distinguish the fundamental principles that determine electron organization, which are usually known by the names of the scientists who discovered or postulated them. The Heisenberg uncertainty principle (B) refers to the inability to know the momentum and position of a single electron simultaneously. The Bohr model (C) was an early attempt to describe the behavior of the single electron in a hydrogen atom. The Rutherford model (D) described a dense, positively charged nucleus. The element shown here, phosphorus, is often used to demonstrate Hund's rule because it contains a half-filled p subshell. Hund's rule explains that electrons fill empty orbitals first before doubling up electrons in the same orbital.

(9-13) Lead is a toxic element that can cause many symptoms, including mental retardation in children. If a body of water is polluted with lead ions at 200 ppb, what is the concentration of lead expressed as molarity? (density of water= 1g/mol and ppb= grams per 10^9 grams of solution) A) 9.7x10^-10 M Pb2+ B) 9.7 x10^-7 M Pb2+ C) 6.2x10^-7 M Pb2+ D) 6.2x10^-6 M Pb2+

B 200 ppb of Pb2+ is equivalent to 200 grams of Pb2+ in 10^9 grams of solution; given the extremely low concentration of lead, the mass of the water can be assumed to be approximately 10^9 grams as well. To solve, set up a differential analysis equation. The units needed at the end are moles per liter (molarity), so convert grams of lead to moles of lead and grams of water to liters of water [200gPb2+/10^9gH2O] [10^3gH2O/1L] [1molPb2+/207.2gPb2+] ~ 200x10^3/200x10^9= 1x10^-6M= 10x10^-7M

(7-12) Which of the following devices would be the most appropriate to use to measure the heat capacity of a liquid? A) thermometer B) calorimeter C) barometer D) volumetric flask

B A calorimeter measures heat transfer. Although calorimeters often incorporate thermometers, the thermometer itself only tracks temperature, not heat transfer, so A is incorrect. C is irrelevant; barometers measure changes in pressure. D is also incorrect because volumetric flasks measure quantities of liquid, not the heat capacity of liquid.

(4-13) In the following reaction: Au2S3 (s) + H2 (g) --> Au (s) + H2S (g) If 2 moles of Au2S3 (s) is reacted with 5 moles of hydrogen gas, what is the limiting reagent? A) Au2S3 (s) B) H2 (g) C) Au (s) D) H2S (g)

B A limiting reagent is by definition a reactant. Because Au and H2S are product, they can be eliminated. Next, the equation needs to be balanced. Two moles of gold (III) sulfide and five moles of hydrogen gas are available. to use up both moles of gold (III) sulfide, 6 moles of hydrogen gas are needed because there is a 1:3 ration between these reactants. Since only 5 moles of hydrogen gas are present, that will have to be the limiting reagent.

(2-5) Antimony is used in some antiparasitic medications- specifically those targeting Leishmania donovani. What type of element is antimony? A) metal B) metalloid C) halogen D) nonmetal

B Antimony (Sb) is on the right side of the periodic table, but not far right enough to be a nonmetal (D). It certainly does not lie far enough to the right to fall into Group VIIA (17) which would classify it as a halogen (C). While sources have rarely classified antimony as a metal (A), it is usually classified as a metalloid (B).

(6-11) Increasing temperature can alter the Keq of a reaction. Why might increasing temperature indefinitely be unfavorable for changing reaction conditions? A) the equilibrium constant has a definite limit that cannot be surpassed B) the products or reactants can decompose at high temperatures C) increasing temperature would decrease pressure, which may or may not alter reaction conditions D) if a reaction is irreversible, its Keq will resist changes in temperature

B At extremely high temperatures, reactants or products may decompose, which will affect the equilibrium and potentially destroy the desired products. A implies that reactions have limits, which is true; however, this does not make increasing temperature unfavorable. C is false because increasing temperature would also increase pressure, assuming constant volume. D is incorrect because it refers to properties of irreversible reactions, which would not be involved in an equilibrium between products and reactants.

(1-7) Suppose an electron falls from n=4 to its ground state, n=1. Which of the following effects is most likely? A) a photon is absorbed B) a photon is emitted C) the electron moves into a p-orbital D) the electron moves into a d-orbital

B Because the electron is moving into the n=1 shell, the only subshell available is the 1s subshell, which eliminates C and D. There will be some energy change, however, as the electron must lose energy to return to the minimum-energy ground state. That will require emitting radiation in the form of a photon.

(9-3) Two organic liquids are combined to form a solution. Based on their structures, will the solution closely obey Raoult's law? (**insert pic here**) A) Yes; the liquids differ due to additional methyl group on toulene and, therefore, will not deviate from Raoult's law B) Yes; the liquids are very similar and, therefore, will not deviate from Raoult's law C) No; the liquids differ due to the additional methyl group on toulene and, therefore, will deviate from Raoult's law D) No; the liquids both contain benzene rings, which will interact with each other and cause deviation from Raoult's law

B Benzene and toulene are both organic liquids and have very similar properties. They are both nonpolar and are almost exactly the same size. Raoult's law states that ideal solution behavior is observed when solute-solute,, solvent-solvent, and solute-solvent interactions are all very similar. Therefore, benzene and toulene in solution will be predicted to behave as a nearly ideal solution.

(5-9) Which of the following best describes the purpose of a catalyst? A) catalysts are used up in the reaction, increasing reaction efficiency B) catalysts increase the rate of the reaction by lowering the activation energy C) catalysts alter the thermodynamics of the reaction to facilitate the formation or products or reactants D) catalysts stabilize the transition state by bringing in to a higher energy

B By definition, a catalyst increases the rate of a reaction by lowering the activation energy, making it easier for both the forward and reverse reactions to overcome this energy barrier. Catalysts are neither used up in the reaction, nor do they alter the equilibrium of a reaction, eliminating A and C. Finally, catalysts stabilize the transition state by lowering its energy, not raising it (D).

(3-1) What is the character of the bond in carbon monoxide? A) ionic B) polar covalent C) nonpolar covalent D) coordinate covalent

B CO has a triple bond between carbon and oxygen, with the carbon and oxygen each retaining one lone pair. In polar covalent bonds, the difference in electronegativity between the bonded atoms is great enough to cause electrons to move disproportionately towards the more electronegative atom but not great enough to transfer electrons completely. This is the case for CO. Oxygen is significantly more electronegative than carbon, so electrons will be disproportionately carried on the oxygen, leaving the carbon atom with a slight positive charge.

(2-12) Of these four atoms, which has the highest electron affinity? (** insert pic here**)

B Electron affinity is related to several factor, including atomic size and filling of the valence shell. As atomic radius increases, the distance between the nucleus and the outermost electrons increases, thereby decreasing the attractive forces between protons and electrons. As a result, increased atomic radius will lead to lower electron affinity. Because atoms are in a low-energy state when their outermost valence electron shell is filled, atoms needing only one or two electrons to complete this shell will have high electron affinities. In this example, B and D need only one more electron to have a noble gas-like electron configuration; because B is smaller, it will have the highest electron affinity.

(11-2) How many electrons are involved in the following half-reaction after it is balanced? Cr2O7^2- + H+ + e- --> Cr2+ + H2O A) 2 B) 8 C) 12 D) 16

B First, balance the atoms in the equation: Cr2O7^2- + 14 H+ --> 2 Cr2+ + 7 H2O Now, adjust the number of electrons to balance the charge. Currently, the left side is a charge of +12. The right side has a charge of +4. To decrease the charge on the left side from +12 to +4, we should add 8 electrons: Cr2O7^2- + 14 H+ + 8e- --> 2 Cr2+ + 7 H2O

(10-2) Which of the following is closest to the pH of a solution containing 5mM H2SO4? A) 1 B) 2 C) 3 D) 4

B First, convert the concentrations to 5x10^-3M. Next, because sulfuric acid is a strong acid, we can assume that, for the majority of sulfuric acid molecules (although not all), both protons will dissociate. The concentration of hydrogen ions is therefore 2x5x10^-3, or 10^-2. The equation for pH is pH=-log[H+]. If [H+]=1-^-2M, then pH=2.

(1-10) Which of the following electronic transitions would result in the greatest gain in energy for a single hydrogen electron? A) an electron moves from n=6 to n=2 B) an electron moves from n=2 to n=6 C) an electron moves from n=3 to n=4 D) an electron moves from n=4 to n=3

B For the electron to gain energy, it must absorb energy from photons to jump up to a higher energy level. There is a bugger jump between n=2 and n=6 than there is between n=3 and n=4.

(9-7) When ammonia, NH3, is used as a solvent, it can form complex ions. For example, dissolving AgCl in Nh3 will result in the complex ion [Ag(Nh30]2+. What effect would the formation of complex ions have on the solubility of a compound like AgCl in NH3? A) The solubility of AgCl will increase because complex ion formation will cause more ions to exist in solution, which interact with AgCl to cause it to dissociate. B) The solubility of AgCl will increase because complex ion formation will consume Ag+ ions and cause the equilibrium to shift away from solid AgCl. C) The solubility of AgCl will decrease because Ag+ ions are in complexes, and the Ag+ ions that are not complexed will associate with Cl- to form solid AgCl. D) The solubility of AgCl will decrease because complex ion formation will consume Ag+ ions and cause the equilibrium to shift toward the solid AgCl.

B Formation of complex ions between silver ions and ammonia will cause more molecules of solid AgCl to dissociate. The equilibrium is driven toward dissociation because the ag+ ions are essentially being removed from solution when they complex with ammonia. This rationale is based upon Le Chatelier's principle, stating that when a chemical equilibrium experiences a change in concentration, the system will shift to counteract that change.

(12-9) Which of the following statements could be true about a Na-Cd cell, based on the information below? Na+ + e- -> Na (E*red=-2.71V) Cd2+ + 2e- -> Cd (E*red=-0.40V) A) it is a galvanic cell, and sodium is the cathode B) it is an electrolytic cell, and cadmium is the anode C) it is a galvanic cell, with E*cell=3.11V D) it is an electrolytic cell, with E*cell=-3.11V

B If this were a galvanic cell, the species with the more positive reduction potential (cadmium) would be reduced. The cathode is always reduced in an electrochemical cell, so sodium would not be the cathode in such a galvanic cell, eliminating A. Sodium would be the cathode in an electrolytic cell, however, which would make cadmium the anode. Thus, the answer is B. Note that we do not have to determine E*cell because we already know the answer. However, the E*cell would be -2.71 - (-0.40) = -2.31V for an electrolytic cell, and +2.31V for a galvanic cell, eliminating C and D.

(2-9) When dissolved in water, which of the following ions is most likely to form a complex ion with H2O? A) Na+ B) Fe2+ C) Cl- D) S2-

B Iron is a transition metal. Transition metals can often form more than one ion. Iron, for example, can be Fe2+ or Fe3+. The transition metals, in these various oxidation states, can often form hydrogen complexes with water. Part of the significance of these complexes is that, when a transition metal can form a complex, its solubility within the related solvent will increase. The other ions given might dissolve readily in water, but because none of them are transition metals, they will not likely form complexes.

(9-12) Reverse osmosis is a process that allows fresh water to be obtained by using pressure to force an impure water source through a semi-permeable membrane that only allows water molecules to pass. What is the minimum pressure that would be required to purify seawater at 25*C that has a total osmolarity of 1000 mOsm/L? A) 23.5 atm B) 24.5 atm C) 24000 atm D) 24500 atm

B Osmotic pressure is given by II= iMRT II= (1000x10^-3 mOsm/L)(0.0812L*atm/mol*K)(298K)~ 1x0.08x300= 24 atm The correct answer must be slightly above the calculated pressure in order for reverse osmosis to proceed.

(12-7) If the surface area of electrode material in an electrochemical cell is tripled, what else is necessarily tripled? I. E*cell II. current III. Keq A) I only B) II only C) I and II only D) II and III only

B Potential, as measured by E*cell, is dependent only on the identity of the electrodes and not the amount present. Similarly, the equilibrium constant depends only on the identity of the electrolyte solutions and the temperature. However, as the electrode material is increased, the surface area participating in oxidation-reduction reactions is increased and more electrons are released, making statement II correct.

(11-11) If a certain metal has multiple oxidation states, its acidity as an oxide generally increases as the oxidation state increases. Therefore, which of the following tungsten compounds is likely to be the strongest acid? A) WO2 B) WO3 C) W2O3 D) W2O5

B Recall that oxygen has an oxidation state of -2. Therefore, in tungsten(IV) oxide (A), tungsten has an oxidation state of +4. In tungsten(VI) oxide (B), it has an oxidation state of +6. In tungsten(III) oxide (C), it is +3. In tungsten pentoxide (D), it is +5.

(12-6) Which of the following compounds is least likely to be found in the salt bridge of a galvanic cell? A) NaCl B) SO3 C) MgSO3 D) NH4NO3

B Salt bridges contain inert electrolytes. Ionic compounds, such as A, C, and D, are known to be strong electrolytes because they completely dissociate in solution. B cannot be considered an electrolyte because its atoms are covalently bonded and will not dissociate in aqueous solution. B and C may appear similar, but there is an important distinction to be made. C implies that Mg2+ and SO3^2- are the final, dissociated ionic constituents, while B implies that neutral SO3 would have to be dissolved in solution.

(10-4) Which of the following bases is the weakest? A) KOH B) NH3 C) CH3NH2 D) Ca(OH)2

B Soluble hydroxides of Group IA and IIA metals are strong bases, eliminating A and D. B and C are both weak bases, however, methylamine contains an alkyl group, which is electron-donating. This increases the electron density on the nitrogen in methylamine, making it a stronger (Lewis) base. Therefore, ammonia is the weakest base.

(6-12) Which of the following is true of equilibrium reactions? I. an increase in k1 results in a decrease in k-1 II. as the concentration of products increases, the concentrations of reactants decreases III. the equilibrium constant is altered by changes in temperature A) I only B) II and III only C) I and III only D) I, II, and III

B Statement I is false because the addition of a catalyst could increase the rate constants of both the forward and reverse reactions. Statement II is true because- for products to come into existence- reactants must be used up. Statement III is also true: all K values are temperature- dependent.

(5-12) (**insert pic here**) A) the first step of the forward reaction B) the first step of the reverse reaction C) the second step of the forward reaction D) the second step of the reverse reaction

B The activation energy of a reaction is represented by the distance on the y-axis from the energy of the reactants to the peak energy prior to formation of products. The activation energy of the first step of the forward reaction, for example, is equal to the distance along the y-axis from point A to point B. The largest energy increase on this graph occurs during the progress from point E to point D, which represents the first step of the reverse reaction.

(1-13) The atomic weight of hydrogen is 1.008 amu. What is the percent composition of hydrogen by isotope, assuming that hydrogen's only isotopes are 1H and 2D? A) 92% H, 8% D B) 99.2% H, 0.8% D C) 99.92% H, 0.08% D D) 99.992% H, 0.008% D

B The easiest way to approach this problem is to set up a system of two algebraic equations, where H and D are the percentages of H (mass= 1 amu) and D (mass= 2 amu), respectively. Your setup should look like the following system: H+D= 1 (percent H+ percent D= 100%) 1H+2D= 1.008 (atomic weight calculation) Rearranging the first equation and substituting into the second yields (1-D) +2D= 1.008, or D= 0.008. This is 0.8%, so there is .8% D.

(4-12) In the process of photosynthesis, CO2 and water combine with energy to form glucose and oxygen, according to the following equation: CO2 + H2O --hv--> C6H12O6 + O2 What is the theoretical yield of glucose if 30 grams of water are reacted with excess carbon dioxide and energy, according to the equation above? A) 30.0 g B) 50.0 g C) 300.1 g D) 1801 g

B The equation given it unbalanced, so the first step is to balance it. The theoretical yield is the amount of product synthesized if the limiting reagent is completely used up. This question therefore asks how much glucose is produced if the limiting reagent is 30g of water. Mass of glucose produced= 50g C6H12O6.

(6-9) Which of the following actions does not affect the equilibrium position of a reaction? A) adding or removing heat B) adding or removing a catalyst C) increasing or decreasing concentrations of reactants D) increasing or decreasing volumes of reactants

B The equilibrium of a reaction can be changes by several factors. Adding or subtracting heat (A) would shift the equilibrium based on the enthalpy change of the reaction. Increasing reactant concentrations would shift the equilibrium in the direction of the product, and the opposite would occur if reactant concentrations were decreased, eliminating C. Changing the volume of a reactant would affect any reaction with gaseous reactants or products, eliminating D. while adding or removing a catalyst would change the reaction rates, it would not change where the equilibrium lies.

(8-4) A 0.040 g piece of magnesium is placed in a beaker of hydrochloric acid. Hydrogen gas is generated according to the following equation: Mg (s) + 2 HCl (aq) --> MgCl2 (aq) + H2 (g) The gas is collected over water at 25*C and the gauge pressure during the experiment reads 748 mmHg. The gas displaces a volume of 100 mL. The vapor pressure of water at 25*C is approximately 24.0 mmHg. Based on this data, how many moles of hydrogen are produced in this reaction? (R=0.0821 L*atm/mol*K =8.314 J/K*mol) A) 4.04x10^-5 moles hydrogen B) 4.09x10^-3 moles hydrogen C) 3.07x10^-2 moles hydrogen D) 3.11 moles hydrogen

B The pressure of the gas is calculated by subtracting the vapor pressure of water from the measured pressure during the experiment: 784 mmHg- 24 mmHg= 760 mmHg, or 1 atm. This is because the reaction is carried out in an aqueous environment; the water present will contribute to the partial pressure of the gas over the liquid. The ideal gas law can be used to calculate the moles of hydrogen gas. The volume of the gas is 0.100 L, the temperature is 298 L, and R=0.0821 L*atm/mol*K. n= PV/RT= (1 atm)(0.1L)/(0.821L*atm/mol*K)(298K)~ 1/ .8x300= 1/240 ~1/250= 0.004.

(10-5) The function of a buffer is to: A) maintain a neutral pH B) resist changes in pH when small amounts of acid or base are added C) slow down reactions between acids and bases D) speed up reactions between acids and bases

B The purpose of a buffer is to resists changes in the pH of a reaction. Buffers are not generally used to affect the kinetics of a reaction, so C and D are incorrect. A is correct only in specific circumstances where the pH of the buffer solution itself is neutral. Many natural buffer systems maintain pH in the acidic or basic ranges.

(10-9) (**insert pic here**) What is the approximate value of pKa2? A) 3.6 B) 4.1 C) 5.5 D) 7.2

B The value of the second pKa is found at the midpoint between the first and second equivalence points. In this curve, that corresponds to pH=4.1. Just like the first pKa, it is in the center of a flat buffering region.

(3-15) Which of the following best describes an important property of bond energy? A) bond energy increases with increasing bond length B) the more shared electron pairs comprising a bond, the higher the energy of that bond C) single bonds are more difficult to break than double bonds D) bond energy and bond length are unrelated

B This answer requires an understanding of the trends that cause higher or lower bond energies. Bonds of high energy are those that are difficult to break. These bonds tend to have more shared pairs of electrons and, thus, cause a stronger attraction between the two atoms in the bonds. This stronger attraction also means that the bond length of a high-energy, high-order bond such as a triple bond is shorter than that of its lower-energy counterparts such as single or double bonds.

(2-8) Which of the following is an important property of elements in group 2 of the periodic table? A) these elements are the best electrical conductors in the periodic table B) these elements form divalent cations C) the second ionization energy for these elements is lower than the first ionization energy D) the atomic radii of these elements decrease as one moves down the column

B This block represents the alkaline earth metals, which form divalent cations, or ions with a +2 charge. all of the elements in Group IIA have two electrons in their outermost s subshell. Because loss of these two electrons would leave a full octet as the outermost shell, becoming a divalent cation is a stable configuration for all of the alkaline earth metals. Although some of these elements might be great conductors, they are not as effective as the alkali metals. C is also incorrect because, although forming a divalent cation is a stable configuration for the alkaline earth metals, the second ionization energy is still always higher than the first. Finally, D is incorrect because atomic radii increase when moving down a group of elements because the number of electron shells increases.

(4-11) A reaction that utilizes oxygen and hydrocarbons as reactants and that produces carbon dioxide and water as products is best characterized as: A) single-displacement B) combustion C) metathesis D) decomposition

B This description characterizes a combustion reaction because a hydrocarbon acts as a fuel when reacting with oxygen. CO2 (an oxide) and water are the products of such a reaction.

(3-2) Which of the following molecules contains the oxygen atoms with the most negative formal charge? A) H2O B) CO3^2- C) O3 D) CH2O

B To answer this question, one must understand the contribution of resonance structures to average formal charge. In B, there are three possible resonance structures. Each of the three oxygen atoms carries a formal charge of -1 in two out of the three structures. This averages to approximately -2/3 charge on each oxygen atom, which is more negative than in the other choices. Both water and formaldehyde (A and D) have no formal charge on the oxygen. Ozone (C) has a -1/2 on two of the three oxygens and a +1 charge on the central oxygen.

(12-2) Given the following standard reduction potentials: Zn2+ + 2e- --> Zn (E*red= -0.763V) Ag+ + e- --> Ag (E*red= +0.337V) What is the standard electromotive force of the following reaction? Zn2+ + 2 Ag --> 2 Ag+ + Zn A) -2.2 V B) -1.1 V C) ++1.1 V D) +2.2 V

B To determine the standard electromotive force of a cell, simply subtract the standard reduction potentials of the two electrodes. In this case, the cathode is zinc because it is being reduced; The anode is silver because it is being oxidized. Thus, E*cell= E*red,cathode - E*red, anode= -0.763 - 0.337 = -1.10V While we must multiply the silver half-reaction by two to balance electrons, the actual value for the reduction potential does not change. Remember that the standard reduction potential is determined by the identity of the electrode, not the amount of it present.

(8-12) A gas at a temperature of 27*C has a volume of 60.0 mL. What temperature change is needed to increase this gas to a volume of 90.0 mL? A) a reduction of 150*C B) an increase of 150*C C) a reduction of 13.5*C D) an increase of 13.5*C

B Using Charles's law, first we must convert the temperature to kelvins by adding 273 to get 300K as the initial temperature. Think of this as a proportionality: If the volume is multiplied by 3/2, the temperature will also have to be multiplied by 3/2. Thus the final temperature is 450K, which represents a 150K increase.

(1-15) Which of the following species is represented by the electron configuration 1s2 2s2 2p6 3s2 2p6 4s1 3d5? I. Cr II. Mn+ III. Fe2+ A) I only B) I and II only C) II and III only D) I, II, and III

B When dealing with ions, you cannot directly approach electronic configurations based on the number of electrons they currently hold. First examine the neutral atom's configuration, and then determine which electrons are removed. Cr: [Ar] 4s1 3d5 Mn+: [Ar] 4s1 3d5 Fe2+: [Ar] 4s0 3d6

(7-7) For a certain chemical process, deltaG*= -4.955 kJ/mol. What is the equilibrium constant Keq for this reaction? (R= 8.314 J/mol*K) A) Keq=1.0 B) Keq= 7.4 C) Keq= 8.9 D) Keq= 10

B deltaG*rxn= -RTlnKeq deltaG*rxn is -4.955 kJ/mol, R is 8.314 J/mol*K, and T is 298 K because the reaction is occurring under standard conditions. Because deltaG*rxn uses kilojoules in its units and R uses joules, one needs to be converted. -4955 J/mol= -(8.314 J/mol*K)(298K)(lnKeq) lnKeq= 4955 J/mol / (8.314 J/mol*K)(298K) ~ 5000/ 8x300 = 5000/2400 ~ 5000/25000= 2 If lnKeq=2, then Keq=e^2. the value of e is approximately 2.7, so e^2= 2.7^2 will be a number between 2^2=4 and 3^2-9. Both B and C fit this, but 8.9 is very close to 9, so we can assume that its square root is very very close to 3. The answer should be a little smaller, so 7.4 (B) is correct.

(9-11) Which of the following will cause the greatest increase in the boiling point of water when it is dissolved in 1.00 kg H2O? A) 0.4 mol calcium sulfate B) 0.5 mol iron(III) nitrate C) 1.0 mol acetic acid D) 1.0 mol sucrose

B deltaTb=iKbm. m is the molality of the solution and Kb is the boiling point elevation constant. In this case, the solvent is always water, so Kb will be the same for each solution The number of dissociated particles from each of the original species is needed- this is referred to as the van't Hoff factor (i) and is multiplied by molality to give a normality (the concentration of the species of interest- in this case, all particles). (**insert pic here**) The choice is between iron(III) nitrate and acetic acid. The fact that acetic acid is a weak acid indicates that only a few particles will dissociate into H+ and acetate. Therefore, the normality of the acetic acid will be much closer to 1.0 than 2.0.

(3-10) Although the octet rule dictates much of molecular structure, some atoms can violate the octet rule by being surrounded by more than 8 electrons. Which of the following is the best explanation for why some atoms can exceed the octet? A) atoms that exceed the octet already have eight electrons in their outermost shell B) atoms that exceed the octet only do so when bonding with transition metals C) atoms that exceed the octet can do so because they have d-orbitals in which extra electrons can reside D) some atoms can exceed the octet because they are highly electronegative

C All atoms in the third period or greater have d-orbitals, which can hold an additional 10 electrons. The typical "octet" electrons reside in s- and p-orbitals, but elements in period 3 or higher can place electrons into these d-orbitals.

(3-11) Which of the following intermolecular forces provides the most accurate explanation for why noble gases can liquefy? A) hydrogen bonding B) ion-dipole interactions C) dispersion forces D) dipole-dipole interactions

C All of the listed types of forces describe interactions between different types of molecules. However, noble gases are entirely uncharged and do not have polar covalent bonds, ionic bonds, or dipole moments. Therefore, the only intermolecular forces experienced by noble gases are London dispersion forces. Although these interactions are small in magnitude, they are necessary for condensation into a liquid.

(12-13) An electrolytic cell necessarily has: A) deltaS*>0 B) deltaG*<0 C) Keq<1 D) E*cell>0

C An electrolytic cell is nonspontaneous. Therefore, the deltaG* must be positive and E*cell must be negative, eliminating B and D. The change in entropy may be positive or negative, depending on the species involved, eliminating A.

(8-8) The plots of two gases at STP are shown below. One of the gases is 1.0L of helium and the other in 1.0L of bromine. Which plot corresponds to each gas and why? (**insert pic here**) A) curve A is helium and curve B is bromine because helium has a smaller molar mass than bromine B) curve A is helium and curve B is bromine because the average kinetic energy of bromine is greater than the average kinetic energy of helium C) curve A is bromine and curve B is helium because helium has a smaller molar mass than bromine D) curve A is bromine and curve B is helium because the average kinetic energy of bromine is greater than the average kinetic energy of helium

C At STP, the difference between the distribution of speeds for helium and bromine gas is due to the difference in molar mass. Helium has a smaller molar mass than bromine. Particles with small masses travel faster than those with large masses, so the helium gas corresponds to curve B, which has a higher average speed. Because the gases are at the same temperature (273 K), they have the same average kinetic energy, eliminating B and D.

(2-3) What determines the length of an element's atomic radius? I. The number of valence electrons II. The number of electron shells III. The number of neutrons in the nucleus A) I only B) II only C) I and II only D) I, II, and III

C Atomic radius is determined by multiple factors. Of the choices given, the number of valence electrons does have an impact on the atomic radium. As one moves across a period (row), protons and valence electrons are added, and the electrons are more strongly attracted to the central protons. This attraction tightens the atom, shrinking the atomic radius. The number of electron shells is also significant, as demonstrated by the trend when moving down a group (column). As more electron shells are added that separate the positively charged nucleus from the outermost electrons, the electrostatic forces are weakened, and the atomic radius increases. The number of neutrons is irrelevant because it does not impact these attractive forces.

(9-6) The entropy change when a solution forms can be expressed by the term deltaS*soln. When water molecules become ordered around an ion as it dissolves, the ordering would be expected to make a negative contribution to deltaS*soln. An ion that has more change density will have a greater hydration effect, or ordering of water molecules. Based on this information, which of the following compounds will have the most negative contribution to deltaS*soln? A) KCl B) LiF C) CaS D) NaCl

C CaS will cause the most negative contribution to deltaS*soln through hydration effects because the Ca2+ and S2- ions have the highest charge density compared to the other ions. All of the other ions have charges of +1 or -1, whereas Ca2+ and S2- each have charges with a magnitude of 2.

(12-15) Which of the following best describes why overcharging a Ni-Cd battery is not detrimental? A) The energy density of a Ni-Cd battery is high, so it can store more charge than other batteries per its mass B) The electrodes of a Ni-Cd battery can discharge through the circuit when they are fully charged C) The Ni-Cd battery will stop accepting electrons from an outside source when its electrodes are recharged D) Ni-Cd batteries have a high surge current and can dissipate the overcharge before damage can occur to electrodes

C During the recharge cycle, Ni-Cd cells will accept current from an outside source until the Cd and NiO(OH) electrodes are pure; at this point, the reaction will stop because Cd(OH)2 runs out and no more electrons can be accepted. A and B are both true, but they fail to explain why overcharging the battery is not a problem with N-Cd batteries. Finally, surge current refers to the initial burst of current seen in some batteries; once charged, the surge current will not increase even if the power source continues to be run because no additional charge will be stored on the electrodes, eliminating D.

(3-8) Which of the following is the best explanation of the phenomenon of hydrogen bonding? A) hydrogen has a strong affinity for holding onto valence electrons B) hydrogen can only hold two valence electrons C) electronegative atoms disproportionately carry shared electron pairs when bonded to hydrogen D) hydrogen bonds have ionic character

C Electronegative atoms bonded to hydrogen disproportionately pull covalently bonded electrons toward themselves, which leaves hydrogen with a partial positive character. That partial positive charge is attracted to nearby negative or partial negative charges, such as those on other electronegative atoms.

(2-6) The properties of atoms can be predicted, to some extent, by their location within the periodic table. Which property or properties increases in the direction of the arrows shown? ^^[]>> I. electronegativity II. atomic radius III. first ionization energy A) I only B) I and II only C) I and III only D) II and III only

C Electronegativity describes how strong an attraction an element will have for electrons in a bond. A nucleus with a larger effective nuclear charge will have a higher electronegativity; Zeff increases toward the right side of a period. A stronger nuclear pull will also lead to increased first ionization energy, as the forces make it more difficult to remove an electron. The vertical arrow can be explained by the size of the atoms. As size decreases, the positive charge becomes more effective at attracting electrons in a chemical bond (higher electronegativity), and the energy required to remove an electron (ionization energy) increases.

(11-15) An assay if performed to determine the gold content in a supply of crushed ore. One method for pulling gold out of ore is to react it in a concentrated cyanide (CN-) solution. Au + NaCN + O2 + H2O --> Na[Au(CN)2] + NaOH An indicator is used during this reaction, and approximately 100 mL of a 2M NaCN solution is used to reach the endpoint. How many moles of Au are present in the crushed ore? A) 0.01 mol B) 0.02 mol C) 0.10 mol D) 0.20 mol

C First, balance the chemical equation: 4 Au + 8 NaCN + O2 + 2 H2O -> 4 Na[Au(CN)2] + 4 NaOH Now, determine the number of moles of NaCNused in the reaction: 0.1Lx2mol/L= 0.2 mol NaCN If 0.2 mol NaCN are used in the reaction, then 0.2 mol NaCNx 4molAu/8molNaCN= 0.1 mol Au is oxidized.

(1-3) What is the maximum number of electrons allowed in a single atomic energy level in terms of the principal quantum number n? A) 2n B) 2n+2 C) 2n^2 D) 2n^2 +2

C For any value of n, there will be a maximum of 2n^2 electrons; two per orbital. This can also be determined from the periodic table. There are only two elements (H and He) that have valence electrons in the n=1 shell. Eight elements (Li to Ne) have valence electrons in the n=2 shell. This is the only equation that matches this pattern.

(7-14) Which of the following phase changes is associated with the largest decrease in entropy? A) fusion B) solidification C) deposition D) sublimation

C Gases have the highest entropy, and solids have the lowest. Therefore, a phase change from a gas to a solid- deposition- would have the largest decrease in entropy of any phase change process.

(11-4) What is the oxidation number of chlorine in NaClO? A) -1 B) 0 C) +1 D) +2

C In NaClO (sodium hypochlorite), sodium carries its typical +1 charge, and oxygen carries its typical -2 charge. This means that the chlorine atom must carry a +1 charge in order to balance the overall charge of 0.

(12-4) An electrolytic cell is filled with water. Which of the following will move toward the cathode of such a cell? I. H+ ions II. O2- ions III. electrons A) I only B) II only C) I and III only D) II and III only

C In an electrolytic cell, ionic compounds are broken up into their constituents; The cations (positively charged ions) migrate toward the cathode, and the anions (negatively charged ions) migrate toward the anode. In this case, the cations are H+ ions (protons), so option I is correct. Electrons flow from anode to cathode in all types of cells, meaning the option III is also correct. Option II is incorrect for two reasons. First, it is unlikely that the anions in any cell would be O2- rather than OH-. Second, and more significantly, these anions would flow to the anode, not the cathode.

(6-13) Compound A has a Ka (equilibrium constant of acid dissociation) of approximately 10^-4. Which of the following compounds is most likely to react with a solution of compound A? A) HNO3 B) NO2 C) NH3 D) N2O5

C Ka is equal to the ration of products to reactants, with each species raised to its stoichiometric coefficient. A compound with a Ka greater than 10^-7 contains more H+ cations than HA- anions at equilibrium, which makes it an acid. This means that the compound in question is likely to react with a compound that is basic. Of the answer choices, NH3 is the only base.

(10-14) What is the approximate pH of a 1.2x10^-5M aqueous solution of NaOH? A) 4.92 B) 7.50 C) 9.08 D) 12.45

C NaOH is a strong base; as such, there will be 1.2x10^-5 M OH- in solution. Based on this information alone, the pOH must be between 4 and 5, and the pH must be between 9 and 10. Using the shortcut, pOH~5-0.12= 4.88. pH= 14-pOH= 9.12.

(11-13) Potentiometry in an oxidation-reduction titration is analogous to performing an acid-base titration with a: A) acidic indicator B) basic indicator C) pH meter D) oxidizing agent

C Potentiometry refers to carrying out an oxidation-reduction titration with a voltmeter present to get precise readings of the reaction's electromotive force (emf) to determine the end point. This is analogous to using a pH meter in an acid-base titration because it uses technology to get precise readings for plotting a titration curve. Indicators, as in A&B, can be used in both acid-base in redox titrations, but provide a qualitative rather than quantitative analysis of the titration. Oxidizing and reducing agents are used in redox titrations, not acid-base titrations, eliminating D.

(6-14) consider the following two reaction: 3 A + 2 B <--> 3 C + 4 D (reaction 1) 4 D + 3 C <--> 3 A + 2 B (reaction 2) If Keq for reaction 1 is equal to 0.1, what if Keq for reaction 2? A) 0.1 B) 1 C) 10 D) 100

C Reaction 2 is the reverse of reaction 1. This means that Keq for reaction 2 is the inverse of Keq of reaction 1, so the answer is 1/0.1 = 1/ 1/10 = 10.

(7-6) At standard temperature and pressure, a chemical process is at equilibrium. What is the free energy of reaction (deltaG) for this process? A) deltaG>0 B) deltaG<0 C) deltaG=0 D) there is not enough information to determine the free energy of the reaction

C Standard temperature and pressure indicate 0*C and 1 atm. Gibbs free energy is temperature dependent, but is a reaction is at equilibrium, deltaG=0.

(11-10) In the compound KH2PO4, which element has the highest oxidation number? A) K B) H C) P D) O

C Start with the atoms that have oxidation states of which you are certain. Potassium is a Group 1A metal, and therefore must have an oxidation state of +1. Hydrogen is almost always +1, unless it is paired with a less electronegative element, which is not the case here. Oxygen is generally -2. Because there are four oxygens, they create a total negative charge of -8 which is partially balanced by two hydrogens (+2) and potassium (+1). Therefore, phosphorus has a + 5 charge, making it the highest oxidation state.

(3-5) Both CO3^2- and ClF3 have three atoms bonded to a central atom. what is the best explanation for why CO3^2- has trigonal planar electronic geometry, while ClF3 has trigonal bipyramidal electronic geometry? A) CO3^2- has multiple resonance structures, while ClF3 does not B) CO3^2- has a charge of -2 while ClF3 has no charge C) ClF3 has lone pairs on its central atom, while CO3^2- has none D) CO3^2- has lone pairs on its central atom, while ClF3 has none

C The central carbon in carbonate has no lone pairs. It has three resonance structures, each of which involves a double bond between carbon and one of the three oxygens. Having made four bonds, carbon has no further orbitals for bonding or to carry lone pairs. This makes carbonate's geometry trigonal planar. Alternatively, Clf3 also has three bonds; however, chloride still maintains two extra lone pairs. These lone pairs each inhabit one orbital, meaning that the central chloride must organize five items about itself; three bonds to fluorides and two lone pairs. The best configuration for maximizing the distance between all of these groups is trigonal bipyramidal. A and B are true statements but do not account for the difference in geometry.

(8-10) Given that the gases at the center of the sun have an average molar mass of 2.00 g/cm^3, compressed to a density of 1.20 g/cm^3 under 1.30x10^9 atm of pressure, what is the temperature at the center of the sun? A) 2.6x10^4K B) 2.6x10^6K C) 2.6x10^7K D) 2.6x10^10K

C The ideal gas law can be modified to include density because the number of moles of gas (n) is equal to the mass divided by the molar mass. T= PM/densityR= 2.6x10^7K

(1-9) Which of the following best explains the inability to measure and position the momentum exactly and simultaneously according to the Heisenberg uncertainty principle? A) imprecision in the definition of the meter and kilogram B) limits on accuracy of existing scientific instruments C) error in one variable is increased by attempts to measure the other D) discrepancies between the masses of nuclei and of their component particles

C The limitations placed by the Heisenberg uncertainty principle are caused by limitations inherent in the measuring process: if a particle is moving, it has momentum, but trying to measure that momentum necessarily creates uncertainty in the position. Even if we had an exact definition of the meter (A), or perfect measuring devices (B), we still wouldn't be able to measure position and momentum simultaneously and exactly.

(5-10) If the rate law for a reaction is: rate= k[A]0[B]2[C]1 What is the overall order of the reaction? A) 0 B) 2 C) 3 D) 4

C The overall order of a reaction is the sum of individual orders in the reaction. Therefore, the rate order is 0+2+1=3.

(3-13) Which of the following is the best name for the new bond formed in the reaction shown? (**insert pic here**) A) nonpolar covalent bond B) ionic bond C) coordinate covalent bond D) hydrogen bond

C The reaction in this question shows a water molecule, which has two lone pairs of electrons on the central oxygen, combining with a free hydrogen cation. The resulting molecule, H3O+, has formed a new bond between H+ and H2O. This bond is created via the sharing of one of oxygen's lone pairs with the free H+ ion. This represents the donation of a shared pair of electrons from a Lewis base (H2O) to a Lewis acid (H+, electron acceptor). This type of bond is also called a coordinate covalent bond.

(3-3) Which of the following structures contributes most to NO2's resonance hybrid? (**insert pic here**) A) I only B) III only C) I and II only D) I, II, and III

C The two greatest contributors are structures I and II. Resonance structures are representations of how charges are shared across a molecule. In reality, the charge distribution is a weighted average of contributing resonance structures. The most stable resonance structures are those that minimize charge on the atoms in the molecule; the more stable the structure, the more it will contribute to the overall charge distribution in the molecule. Structures I and II minimize formal charges, so will be the largest contributors to the resonance hybrid.

(11-7) One way to test for the presence of iron in solution is by adding potassium thiocyanate to the solution. The product when this reagent reacts with iron is FeSCN2+, which creates a dark red color in solution via the following net ionic equation: Fe3+ + SCN- --> FeSCN2+ A) 110g B) 220g C) 400g D) 500g

C What you are showing is a net ionic equation. If 2 moles of FeSCN are created, 2 moles of Fe3+ must be used because the more ratio is 1:1. Iron sulfate has the formula Fe2(SO4)3 because sulfate has a charge of -2 and iron has a charge of +3 (based on the net ionic equation). Therefore, one mole of iron sulfate is needed to make 2 moles of iron for the reaction. The molar mass of iron sulfate is 2x55.8g/mol + 3x32.1g/mol+ 12x16.0g/mol= 399.9g/mol This most closely matches answer C. The most common error would be to calculate the amount of iron, which would be 111.6 g (A).

(11-1) Consider the following: 6 Na (s) + 2 NH3 (aq) --> 2 Na3N (s) + 3 H2 (g) Which species acts as an oxidizing agent? A) Na B) N in NH3 C) H in NH3 D) H2

C The oxidizing agent is the species that is reduced in any given equation. In this problem, 6 hydrogen atoms with +1 oxidation states in NH3 are reduced to three neutral H2 molecules.

(5-2) In a certain equilibrium process, the activation energy of a forward reaction is greater than the activation of the reverse reaction. This reaction is: A) endothermic B) exothermic C) spontaneous D) nonspontaneous

D (**insert pic here**) If the activation energy of the forward reaction is greater than the activation energy of the reverse reaction, then the products must have a higher free energy than the reactants. The overall energy of the system is higher at the end that it was in the beginning. The net free energy change is positive, indicating an endergonic (nonspontaneous) reaction. Endothermic and exothermic are associated with enthalpy. While free energy does depend on enthalpy, it also depends on entropy; there is not enough information in the question stem to reliably determine the sign of the entropy change of the reaction.

(10-1) Which of the following is not a Bronsted-Lowry base? (**insert pic here**)

D A Bronsted-Lowry base is defined as a proton acceptor. Ammonia, fluoride, and water each accept a proton. HNO2 is a far better Bronsted-Lowry acid, donating a proton to solution.

(5-3) A reactant in a second-order reaction at a certain temperature is increased by a factor of 4. By how much is the rate of the reaction altered? A) it is unchanged B) it is increased by a factor of 4 C) it is increased by a factor of 16 D) it cannot be determined from the information given

D A second-order reaction can be second-order with respect to one reactant, or first-order with respect to two different reactants. In this case, one reactant was increased by a factor of 4. If the reaction is second-order with respect to this reactant, the rate law will be rate=k[A]1[B]1, and the rate will increase by a factor of 4. We do not know which of these is the correct rate law and, thus, cannot determine the effect on the rate.

(5-11) (**insert pic here**) The overall reaction depicted by this energy diagram is: A) endergonic, because point B is higher than point A B) endergonic, because point C is higher than point A C) exergonic, because point D is higher than point E D) exergonic, because point A is higher than point E

D A system is exergonic if energy is released by the reaction. For exergonic reactions, the net energy change is negative, and the free energy of the final products is lower than the free energy of the initial reactants. Point E, which represents the final products, is lower of the energy diagram than point A, which represents the energy of the initial reactants. Thus, energy must have been given off, and the reaction is exergonic.

(5-1) In a third-order reaction involving two reactants and two products, doubling the concentration of the first reactant causes the rate to increase by a factor of two. What will happen to the rate of this reaction if the concentration of the second reactant is cut in half? A) it will increase by a factor of 2 B) it will increase by a factor of 4 C) it will decrease by a factor of 2 D) it will decrease by a factor of 4

D Based on the information given, the rate is first-order with respect to the concentration of the first reactant; when the concentration of that reactant doubles, the rate also doubles. Because the reaction is third-order, the sum of the exponents in the rate law must be equal to 3. Therefore, the reaction order with respect to the other reactant must be 3-1=2. If the concentration of this second reactant is multiplied by 1/2, the rate will be multiplied by (1/2)^2= 1/4.

(3-7) Despite the fact that both C2H2 and HCN contain triple bonds, the lengths of these triple bonds are not equal. Which of the following is the best explanation for this finding? A) In C2H2, the bond is shorter because it is between atoms of the same element B) The two molecules have different resonance structures C) Carbon is more electronegative than hydrogen D) nitrogen is more electronegative than carbon

D Bond lengths decrease as the bond order increases, and they also decrease with larger differences in electronegativity. In this case, because both C2H2 and HCN have triple bonds, we cannot compare the bond lengths based on bond order. We must then rely on other periodic trends. The bond length decreases when moving to the right along the periodic table's rows because more electronegative atoms have shorter atomic radii. The nitrogen in HCN is likely to hold its electrons closer, or in a shorter radius, than the carbons in C2H2.

(7-4) Which of the following processes has the most exothermic standard heat of combustion? A) combustion of ethane B) combustion of propane C) combustion of n-butane D) combustion of n-pentane

D Combustion often involves the reaction of a hydrocarbon with oxygen to produce carbon dioxide and water. Longer hydrocarbon chains yield greater amounts of combustion products and release more heat in the process- that is, the reaction is more exothermic. Of the hydrocarbons listed here, n-pentane is the longest chain.

(8-2) What is the density of neon gas in g/L at STP? A) 452.3 B) 226.0 C) 1.802 D) 0.9018

D Density=mass/volume. The mass of 1 mole of neon gas= 20.2 grams. At STP, 1 mole of neon occupies 22.4 L. density=20.2g/22.4L ~ 10/11 <1 g/L

(2-10) How many valence electrons are present in elements in the third period? A) 2 B) 3 C) the number decreases as the atomic number increases D) the number increases as the atomic number increases

D Periods refer to rows, and groups refer to columns. Within the same period, an additional valence electron is added with each step toward the right side of the table.

(6-2) What is the equilibrium expression for the reaction Cu2SO4 (s) <--> 2 Cu+ (aq) + SO4^2- (aq)? A) ([Cu+]^2 [SO4^2-]) / [CuSO4] B) (2 x [Cu+] [SO4^2-]) / [Cu2SO4] C) [Cu+] [SO4^2-] D) [Cu+]^2 [SO4^2-]

D Recall that pure solids and liquids do not appear in the equilibrium expression; thus, this Keq has no denominator because the only reactant is a solid, cuprous sulfate. This could also be called Ksp because a solid is dissociating into ions in solution. The correct Keq should have [Cu+] squared because its stoichiometric coefficient is 2.

(7-8) Consider the chemical reaction in the vessel depicted in the following diagram (**insert pic here**) A) the reaction is spontaneous B) the reaction is nonspontaneous C) the reaction is at equilibrium D) there is not enough information to determine if the reaction is spontaneous

D There is not enough information available to determine the free energy of this reaction. While entropy is clearly increasing (there are more particles in the system), it is unclear what the enthalpy change is. Because bonds are breaking, the reaction should be endothermic, meaning that both deltaS and deltaH are positive. In this case, it is a temperature-dependent process, and- without a temperature given- we cannot determine the sign on deltaG.

(11-14) After balancing the following oxidation-reduction reaction, what is the sum of the stoichiometric coefficients of all of the reactants and products? S8 (s) + NO3^- (aq) --> SO3^2- (aq) + NO (g) A) 4 B) 50 C) 91 D) 115

D Utilize the method described earlier to balance this redox reaction. The balanced half-reactions are: S8 + 24 H2O -> 8 SO3^2- + 48 H+ + 32 e- NO3^- + 4 H+ + 3e- -> NO + 2 H2O To get equal numbers of electrons in each half-reaction, the oxidation half-reaction will have to be multiplied by 3, and the reduction half-reaction will have to be multiplied by 32: 3 S8 + 72 H2O -> 24 SO3^2- + 144 H+ + 96 e- 32 NO3^- + 128 H+ + 96 e- -> 32 NO + 64 H2O This makes the overall reaction: 3 S8 + 32 NO3^- + 8 H2O -> 24 SO3^2- + 32 NO + 16 H+ The sum of the stoichiometric coefficients is therefore: 3+32+8+24+32+16= 115

(5-6) What would increasing the concentration of reactants accomplish in a solution containing a saturated catalyst? A) it would increase the rate constant but not the reaction rate B) it would decrease the rate constant but increase the reaction rate C) it would increase the rate constant and increase the reaction rate D) the reaction rate would be unaffected

D While increasing the concentration of reactants can alter the reaction rate in first- or higher-order reactions, saturated solutions containing a catalyst have a maximum turnover rate and cannot increase the rate constant or the reaction rate any higher by adding more reactant molecules.

(6-7) FeI (aq) + I2 (g) --> FeI3 (aq) Which of the following would increase the formation of product? A) decreasing the volume of the container B) decreasing the pressure of the container C) increasing the volume of the container D) decreasing the volume of the container while maintaining a constant pressure

A Both increasing the pressure of the container and decreasing the volume would favor the side with fewer moles of gas, which is the product side. This makes B and C incorrect. D would not disturb the equilibrium- the significance of decreasing the volume of the container in most equilibria is that there is an increase in pressure; in this case, however, the pressure remains constant despite the change in volume.

(9-10) The salt KCl is dissolved in a beaker. To an observer holding the beaker, the solution begins to feel colder as the KCl dissolves. From this observation, one could conclude that: A) deltaS*soln is large enough to overcome the unfavorable deltaH*soln. B) KCl is mostly insoluble in water C) deltaS*soln must be negative when KCl dissolves D) boiling point depression will occur in this solution

A Dissolution is governed by enthalpy and entropy, which are related by deltaG*soln=deltaH*soln-TdeltaS*soln. The cooling of the solution indicates that heat is used up in this bond-breaking reaction. In other words, dissolution is endothermic, and deltaH is positive. the reaction is occurring spontaneously, so deltaG must be negative. The only way that a positive deltaH can result in a negative deltaG is if entropy (deltaS) is a large, positive value (A). Conceptually, that means that the only way to solid can dissolve is if the increase in entropy is great enough to overcome the increase in enthalpy. B is incorrect because it is clearly states in the question stem that KCl dissolves; further, all salts of Group 1 metals are soluble. C is incorrect because deltaS*soln must be positive in order for KCl to dissolve. Finally, D is incorrect because solute dissolution would cause the boiling point to elevate, not depress. It is also not a piece of evidence that could be found simply by observing the beaker's temperature change.

(7-15) Explosions are necessarily characterized by: A) deltaG<0 B) deltaH>0 C) deltaS<0 D) T<0

A In an explosion, a significant amount of heat energy is released, meaning that the reaction is exothermic (deltaH<0), eliminating B. The entropy change associated with an explosion is positive because energy is dispersed over a much larger area, eliminating C. If this is true, the expression deltaH-TdeltaS must be negative, indicating that this is an exergonic process (deltaG<0). Absolute temperature can never be negative, eliminating D.

(3-12) In the structure shown, which atoms have the most positive charge? (**insert pic here**) A) the phosphorous atom has the most positive charge B) all atoms share the charge equally C) the four oxygen atoms share the highest charge D) the oxygen atom at the peak of the trigonal pyramidal geometry has the most positive charge

A In this Lewis diagram, the phosphate molecule has an overall formal charge of -3. The four oxygen atoms would each be assigned a formal charge of -1. Given the overall charge of -3 and the -1 charge on each oxygen, the phosphorus must have a formal charge of +1.

(12-3) Consider the following data: Hg2+ + 2e- --> Hg (E*red=+0.85V) Cu+ + e- --> Cu (E*red=+0.52V) ZN2+ + 3e- --> Al (E*red=-0.76V) Al3+ + 3e- --> Al (E*red=-1.66V) The anode of a certain galvanic cell is composed of copper. Which of the metals from the data table can be used at the cathode, assuming equal concentrations of the two electrolyte solutions? A) Hg B) Cu C) Zn D) Al

A Oxidation occurs at the anode, and reduction occurs at the cathode. Because Cu is the anode, it must be oxidized. The reduction potential of the cathode cannot be less than that of the anode for a galvanic cell. Therefore, mercury (C) must be the cathode. In a concentration cell, the same material is used as both the cathode and anode; However, this question assumes equal concentrations. If both electrolyte solutions have the same concentration, there will be no oxidation-reduction reaction and, therefore, no anode or cathode. This eliminates B.

(1-1) Which of the following is the correct electron configuration for Zn2+? A) 1s2 2s2 2p6 3s2 3p6 4s0 3d10 B) 1s2 2s2 2p6 3s2 2p6 4s2 3d8 C) 1s2 2s2 2p6 3s2 3p6 4s2 3d10 D) 1s2 2s2 2p6 3s2 3p6 4s0 3d8

A Remember that when electrons are removed from al element, forming a cation, they will be removed from the subshell with the highest n value first. Zn0 has 30 electrons, so it would have the configuration of 1s2 2s2 2p6 3s2 3p6 4s2 3d10. The 4s subshell has the highest principle quantum number, so it is emptied first, forming 1s2 2s2 2p6 3s2 3p6 4s0 3d10. Choice B implies that electrons are pulled out of the d orbital (C) and presents the configuration of the uncharged zinc atoms, and D shows the configuration that would exist if four electrons were removed.

(8-7) The kinetic molecular theory states that: A) the average kinetic energy of a molecule of gas is directly proportional to the temperature of the gas in kelvins B) collisions between gas molecules are inelastic C) gas particles occupy discrete areas of space D) all gas molecules have the kinetic energy at the same temperature

A The average kinetic energy is directly proportional to the temperature of a gas in kelvins. The kinetic molecular theory states that collisions between molecules are elastic and thus do not result in a loss of energy, eliminating B. Gas particles are assumed to take up negligible space in kinetic molecular theory, eliminating C. While the average kinetic energy of any gas as a whole is the same at a given temperature, the particles themselves have a distribution of speeds (as seen in the Maxwell-Boltzmann distribution curve), eliminating D.

(9-1) An aqueous solution was prepared by mixing 70g of an unknown nondissociating solute into 100g of water. The solution has a boiling point of 101.0*C. What is the molar mass of the solute? (Kb=0.512K*kg/mol) A) 358.4 g/mol B) 32.3 g/mol C) 123.2 g/mol D) 233.6 g/mol

A The equation deltaTb=iKbm can be used to solve this problem. The change in boiling point is 101.0-100=1.0*C. deltaTb=iKbm --> m= deltaTb/iKb= 1.0K/ (1)(0.512 K*kg/mol) ~2m The van't Hoff factor for this solute is 1 because the molecule does not dissociate into smaller components. Then, convert to grams of solute using the definition of molality: molality= moles of solute/ kg of solvent --> moles of solute= (2m)(0.1kg)= 0.2 mol. The mass is 0.1 kg because 100mL of water has a mass of 0.1 kg. Then, determine the molar mass: molar mass= 70g/0.2 mol= 350 g/mol, which is closest to A.

(10-7) (**insert pic here**) What s the approximate value of pKa1? A) 1.9 B) 2.9 C) 3.8 D) 4.1

A The first pKa in this curve can be estimated by eye. It is located halfway between the starting point (when no base had yet been added) and the first equivalence point (the first steep portion of the graph, around 15mL). This point is at approximately 7-8mL on the x-axis, which corresponds to a pH of approximately 1.9. Notice that this region experiences very little change in pH, which is the defining characteristic of a buffer region.

(11-9) As methanol is converted the methanal, and the methanoic acid, the oxidation number of the carbon: A) increases B) decreases C) increases then decreases D) decreases then increases

A The formula for methanol is H3COH, for methanal is H2CO, and the methanoic acid is HCOOH. If we assign oxidation numbers to carbon in each molecule, it starts at -2, then becomes 0, then becomes +2. In general, it is often easier to think of oxidation as a gain of bonds to oxygen (or a similarly electronegative element), or loss of bonds to hydrogen for organic compounds. Therefore, because the carbon is oxidized as one converts from an alcohol to an aldehyde to a carboxylic acid, the oxidation number must increase.

(1-14) Consider the two sets of quantum numbers shown in the table, which describe two different electrons in the same atom: n l ml ms 2 1 1 +1/2 3 1 -1 +1/2 Which of the following terms best describes these two electrons? A) parallel B) opposite C) antiparallel D) paired

A The terms in the answer choices refer to the magnetic spin of the two electrons. The quantum number ms represents this property as a measure of an electron's intrinsic spin. These electrons' spins are parallel, in that their spins are aligned in the same direction (ms= +1/2 for both species).

(4-8) Using a given mass of KClO3, how would one calculate the mass of oxygen produced in the following reaction, assuming that it goes to completion? 2 KClO3 --> 2 KCl + 3 O2 (**insert pic here**)

A This is a question best answered by dimensional analysis. Keeping in mind that molar mass is measured in grams of a substance per moles of that substance, only A comes out with the units of grams of oxygen. B has the units of grams per mole of oxygen, not grams of oxygen. C has units of moles per gram of oxygen. D has units of mol^2 per gram of oxygen.

(12-11) Which of the following choices is indicative of a spontaneous reaction, assuming standard conditions? A) E*cell is negative B) Q=Keq C) the cell is a concentration cell D) Keq>1

A spontaneous electrochemical reaction has a negative deltaG. Using the equation deltaG*=-RTlnKeq, Keq>1 would result in lnKeq>0, which means deltaG*<0. A negative electromotive force (A) or equilibrium state (B) would not correspond to a spontaneous reaction. Concentration cells can be spontaneous; however, if the concentration cell had reached equilibrium, it would cease to be a spontaneous reaction, eliminating C. When an answer choice may be true, but does not have to be- it is the wrong answer on Test Day.

(6-6) Acetic acid dissociates in solution according to the following equation: CH3COOH <--> CH3COO- + H+ If sodium acetate is added to a solution of acetic acid in excess water, which of the following effects would be observed in the solution? A) decreased pH B) increased pH C) decreased pKeq (pKa) D) increased pKeq (pKa)

B Adding sodium acetate increases the number of acetate ions present. According to Le Chatelier's principle, this change will push this reaction to the left, resulting in a decrease in the number of free H+ ions. Because pH is determined by the hydrogen ion concentration, a decrease in the number of free protons will increase the pH. An acid's Ka (which is the Keq for acid dissociation) will remain constant under a given temperature and pressure, eliminating C and D.

(8-11) The gaseous state of matter is characterized by which of the following properties? I. gases are compressible II. gases assume the volume of their containers III. gas particles exist as diatomic molecules A) I only B) I and II only C) II and III only D) I, II, and III

B Gases are easily compressible because they travel freely with large amounts of space between molecules. Because gas particles are far apart from each other and in rapid motion, they tend to take up the volume of their container. Many gases exist as diatomic molecules, but this is not a property that characterizes all gases, eliminating C.

(10-10) What is the gram equivalent weight of phosphoric acid? A) 24.5 g B) 32.7 g C) 49.0 g D) 98.0 g

B Gram equivalent weight is the weight (in grams) that releases 1 acid of base equivalent from a compound. Because H3PO4 contains 3 protons, we find the gram equivalent weight by dividing the mass of one mole of the species by 3. The molar mass of phosphoric acid is 98 g/mol, so the gram equivalent weight is 32.7g.

(8-5) Ideal gases: I. have no volume II. have particles with no attractive forces between them III. have no mass A) I only B) II only C) I and II only D) I, II, and III

B Ideal gases are said to have no attractive forces between molecules. While each particle within the gas is considered to have negligible volume, ideas gases as a whole certainly do have a measurable volume, thus option I is eliminated. Gases have molar masses, thus option III is eliminated.

(1-2) Which of the following quantum number sets is possible? A) n=2; l=2; ml=1; ms= +1/2 B) n=2; l=1; ml=-1; ms=+1/2 C) n=2; l=0; ml=-1; ms=-1/2 D) n=2; l=0; ml=1; ms=-1/2

B The azimuthal quantum number l cannot be higher than n-1, ruling out A. The ml number, which describes the chemical's magnetic properties, can only be an integer value between -l and l. It cannot be equal to +/-1 if l=0; this would imply that an s orbital has only 3 subshells (-1, 0, and 1) when we know it can only have one. This rules out C and D.

(4-14) Which of the following would make the strongest electrolytic solution? A) a nonpolar covalent compound with significant solubility B) an ionic compound composed of one cation with +3 charge and three anions with -1 charge C) a polar covalent compound with a small dissociation constant D) an ionic compound composed of two cations with +1 charge and one anion with -2 charge

B The best electrolytes dissociate readily (have a high dissociation constant) and are ionic compounds with large amounts of cations and anions. This rules out A and C. D has fewer total ions with a smaller total magnitude of charge and therefore is not as strong an electrolyte as B.

(9-8) One hundred grams of sucrose are dissolved in a cup of hot water at 80*C. The cup of water contains 300.00 mL of water. What is the percent composition by mass of sugar in the resulting solution? (sucrose=C12H22O11, density of water at 80*C=0.975 g/mL) A) 25.0% B) 25.5% C) 33.3% D) 34.2%

B The mass percent of a solute equals the mass of the solute divided by the mass of the total solution times 100%. (100g source) / (300mL H2O)(0.975/mL)+100g sucrose x100% ~ 100/ 300+100 x100%= 100/400 x100%= 25% This answer is slightly lower than the actual value due to rounding, so B is correct.

(8-13) A gaseous mixture contains nitrogen and helium and has a total pressure of 150 torr. The nitrogen particles comprise 80% of the gas and the helium particles make up the other 20%. What is the pressure exerted by each individual gas? A) 100 torr nitrogen, 50.0 torr helium B) 120 torr nitrogen, 30.0 torr helium C) 30.0 torr nitrogen, 120 torr helium D) 50.0 torr nitrogen, 100 torr helium

B The partial pressure of each gas is found by multiplying the total pressure by the mole fraction of the gas. Because 80% of the molecules are nitrogen, the mole fraction of gas is equal to 0.80. Similarly, for helium, the mole fraction is equal to 0.20. To find the pressure exerted by nitrogen, multiply the total pressure (150 torr) by 0.80 to obtain 120 torr of nitrogen. The remainder, 30 torr, is attributable to helium.

(2-1) Lithium and sodium have similar chemical properties. For example, both can form ionic bonds with chloride. Which of the following best explains this similarity? A) both lithium and sodium ions are positively charged B) lithium and sodium are in the same group of the periodic table C) lithium and sodium are in the same period of the periodic table D) both lithium and sodium have low atomic weights

B The periodic table is organized into periods (rows) and groups (columns). Groups are particularly significant because they represent sets of elements with the same valence electron configuration, which in turn will dictate many of the chemical properties of those elements. Although choice A is true, the fact that both ions are positively charged does not explain the similarity in chemical properties; most metals produce positively charged ions. Choice C is not true because lithium and sodium are in the same group, not period. Finally, although lithium and sodium do have relatively low atomic weights, so do several other elements that do not share the same properties, eliminating D.

(1-4) Which of the following equations describes the maximum number of electrons that can fill a subshell? A) 2l+2 B) 4l+2 C) 2l^2 D) 2l^2 +2

B This formula describes the number of electrons in terms of the azimuthal quantum number l, which ranges from 0 to n-1, with n being the principal quantum number. Maximum number of electrons per subshell: subshell; azimuthal quantum number (l); number of electrons s; 0; 2 p; 1; 6 d; 2; 10 f; 3; 14

(2-11) Which of the following elements has the highest electronegativity? A) Mg B) Cl C) Zn D) I

B This question requires knowledge of the trends of electronegativity within the periodic table. Electronegativity increases left to right for the same reasons that effective nuclear charge increases. Electronegativity decreases as you move down the table because there are more electron shells separating the nucleus from the outermost electrons. In this question, chlorine is the furthest toward the top-right corner of the periodic table.

(4-6) What is the most specific characterization of the reaction shown? Ca(OH)2 (aq) + H2SO4 (aq) --> CaSO4 (aq) + H2O (l) A) single-displacement B) neutralization C) double-displacement D) oxidation-reaction

B This reaction is a classic example of a neutralization reaction, in which an acid and a base react to form a salt and usually water. Although this reaction also fits the criteria for a double-displacement reaction, neutralization is a more specific description of the process.

(10-12) A solution is prepared with an unknown concentration of a theoretical compound with a Ka of exactly 1.0. What is the pH of this solution? A) higher than 7 B) exactly 7 C) less than 7 D) there is not enough information to answer the question

C A higher Ka implies a stronger acid. Weak acids usually have a Ka that is several orders of magnitude below 1. The pKa of a compound is the pH at which there are equal concentrations of acid and conjugate base; the pKa of this compound would be -log1=0. With such low pKa, this compound must be an acid. Therefore, the pH of any concentration of this compound must be below 7.

(2-7) Metals are often used for making wires that conduct electricity. Which of the following properties of metals explains why? A) metals are malleable B) metals have low electronegativities C) metals have valence electrons that can move freely D) metals have high melting points

C All four descriptions of metals are true, but the most significant property that contributes to the ability of metals to conduct electricity is the fact that they have valence electrons that can move freely. Malleability is the ability to shape a material with a hammer, which does not play a role in conducting electricity. The low electronegativity and high melting points of metals also do not play a major role in the conduction of electricity.

(6-8) FeI (aq) + I2 (g) --> FeI3 (aq) If this reaction were exothermic, what effect would decreasing the temperature have on the equilibrium? A) the forward reaction rate and the reverse reaction rate both increase B) the forward reaction rate decreases while the reverse reaction rate increases C) the forward reaction rate increases while the reverse reaction rate decreases D) the forward reaction rate and the reverse reaction rate both decrease

C An exothermic reaction produces heat. Decreasing the temperature favors product formation, resulting in an increase in the forward reaction rate with a concomitant decrease in the reverse reaction rate.

(8-14) In which of the following situations is it impossible to predict how the pressure will change for a gas sample? A) the gas is cooled at a constant volume B) the gas is heated at a constant volume C) the gas is heated, and the volume is simultaneously increased D) the gas is cooled, and the volume is simultaneously increased

C Both a change in temperature and a change in volume can affect a gas's pressure. So if one of those two variables is kept constant (A and B), we'll definitely be able to predict which way the pressure will change. At a constant volume, heating the gas will increase its pressure, and cooling the gas will decrease it. What about when both temperature and volume are changing? If both changes have the same effect on pressure, then we can still predict which way it will change. This is the case in D. Cooling the gas and increasing its volume both decrease pressure. C, on the other hand, presents too vague a scenario for us to predict definitely the change in pressure. Heating the gas would amplify the pressure, while increasing the volume would decrease it. Without knowing the magnitude of each influence, it's impossible to saw whether the pressure would increase, decrease, or stay the same.

(7-11) Which of the following statements is true of a process that is spontaneous in the forward direction? A) deltaG>0 and Keq>Q B) deltaG>0 and Keq<Q C) deltaG<0 and Keq>Q D) deltaG<0 and Keq<Q

C For a process to progress forward spontaneously, Q must be less than Keq and will therefore have a tendency to move in the direction toward equilibrium. A spontaneous reaction's free energy is negative by convention.

(12-1) Rusting occurs due to the oxidation-reduction reaction of iron with environmental oxygen: 4 Fe (s) + 3 O2 (g) --> 2 Fe2O3 (s) Some metals cannot react with oxygen in this fashion. Which of the following best explains why iron can? A) iron has a more positive reduction potential than those metals, making it more likely to donate electrons to oxygen B) iron has a more positive reduction potential than those metals, making it more likely to accept electrons from oxygen C) iron has a less positive reduction potential than those metals, making it more likely to donate electrons to oxygen D) iron has a less positive reduction potential than those metals, making it more likely to accept electrons from oxygen

C In the oxidation-reduction reaction of a metal with oxygen, the metal will be oxidized (donate electrons) and oxygen will be reduced (accept electrons). This fact allows us to immediately eliminate B and D. A species with a higher reduction potential is more likely to be reduced, and a species with a lower reduction potential is more likely to be oxidized. Based on the information in the question, iron is oxidized more readily than those metals; this means that iron has a lower reduction potential.

(8-15) Experimenters notice that the molar concentrations of dissolved oxygen in an enclosed water tank has decreased to one-half its original value. In an attempt to counter this decrease, they quadruple the partial pressure of oxygen in the container. what is the final concentration of the gas? A) half the original concentration B) the same as the original concentration C) double the original concentration D) quadruple the original concentration

C Initially the concentration of the gas is decreased to one-half its original value. Recall that concentration (solubility) and partial pressure are directly related- as one increases, the other increases. If the experimenters then quadruple the partial pressure of oxygen in the vessel, the solubility is also increased by a factor of four. One-half times four gives twice the original concentration value. Misreading the answer choices as being related to the concentration before the experimenters increased the partial pressure leads to D.

(2-4) Ionization energy contributes to an atom's chemical reactivity. Which of the following shows an accurate ranking of ionization energies from lowest to highest? A) first ionization energy of Be< second ionization energy of Be< first ionization energy of Li B) first ionization energy of Be< first ionization energy of Li< second ionization energy of Be C) first ionization energy of Li< first ionization energy of Be< second ionization energy of Be D) first ionization energy of Li< second ionization energy of Be< second ionization energy of Be< first ionization energy of Be

C Ionization energy increases from left to right, so the first ionization energy of lithium is lower than that of beryllium. Second ionization energy is always larger than first ionization energy, so beryllium's second ionization energy should be the highest value. This is because removing an additional electron from Be+ requires one to overcome a significantly larger electrostatic force.

(9-4) Which of the following explanations best describes the mechanism by which solute particles affect the melting point of ice? A) melting point is elevated because the kinetic energy of the substance increases B) melting point is elevated because the kinetic energy of the substance decreases C) melting point is depressed because solute particles interfere with lattice formation D) melting point is depressed because solute particles enhance lattice formation

C Melting point depresses upon solute addition, making A and B incorrect. Solute particles interfere with lattice formation, the highly organized state in which solid molecules align themselves. Colder-than-normal conditions are necessary to create the solid structure.

(3-14) Both BF3 and NH3 have three atoms bonded to the central atom. Which of the following is the best explanation for why the geometry of these two molecules is difference? A) BF3 has three bonded atoms and no lone pairs, which makes its geometry trigonal pyramidal B) NH3 is nonpolar, while BF3 is polar C) NH3 has three bonded atoms and one lone pair, which makes its geometry trigonal pyramidal D) BF3 is nonpolar, while NH3 is polar

C NH3 has three hydrogen atoms bonded to the central nitrogen, which also has a lone pair. These four groups- three atoms, one lone pair- lead NH3 to assume tetrahedral electronic geometry yet trigonal pyramidal molecular geometry. The nitrogen in ammonia is sp3-hybridized. By hybridizing all three p-orbitals and the one s-orbital, four groups are arranged about the central atom, maximizing the distances between the groups to minimize the energy of the molecule with a tetrahedral configuration. In contrast, BF3 has three atoms and no lone pairs, resulting in sp2-hybridization. Its shape is called trigonal planar.

(3-9) Which of the following best describes the number and character of the bonds in an ammonium cation? A) three polar covalent bonds B) four polar covalent bonds, of which none are coordinate covalent bonds C) four polar covalent bonds, of which one is a coordinate covalent bond D) four polar covalent bonds, of which two are coordinate covalent bonds

C Recall that ammonium is NH4+ while ammonia is NH3. Ammonium is formed by the association of NH#, an uncharged molecule with a lone pair on the nitrogen, with a positively charged hydrogen cation. In other words, NH3 is a Lewis base, while H+ is a Lewis acid. This type of bonding between a Lewis acid and base is a coordinate covalent bond.

(4-4) Which of the following molecules cannot be expressed by the empirical formula CH? (**insert pic here**)

C The definition of an empirical formula is a formula that represents a molecule with the simplest ration, in whole numbers, of the elements comprising the compound. In this case, given the empirical formula CH, any molecule with carbon and hydrogen atoms in a 1:1 ration would be accurately represented by this empirical formula. C has three carbon atoms and four hydrogen atoms. Both its molecular and empirical formulas would be C3H4 because this formula represents the smallest whole-number ration of its constituent elements.

(5-8) The following data shown in the table were collected for the combustion of the theoretical compound XH4: XH4 + 2O2 --> XO2 + 2 H2O (**insert pic here**) A) rate=k[XH4][O2] B) rate=k[XH4][O2]2 C) rate=k[XH4]2[O2] D) rate=k[XH4]2[O2]2

C The generic rate law is rate=k[XH4]^x[O2]^y. In the first two trials, the concentration of XH4 is held constant while the concentration of O2 is multiplied by 4, and the rate of the reaction also increases by a factor of approximately 4. This gives the proportion change in rate= [O2]^y, or 4=4^y, meaning y=1. The rate law can be updated to rate=k[XH4]^x[O2]^1. In the last two trials, the concentration of O2 is held constant while the concentration of XH4 is doubled, and the rate of the reaction is increased by a factor of approximately 4. This gives change in rate=[XH4]^x, or 4=2^x and x=2. The rate law can be updated to rate=k[XH4]^2[O2]^1. The final version of the rate law is rate=k[XH4]^2[O2].

(10-8) (**insert pic here**) At what pH is the second equivalence point? A) pH= 3.0 B) pH= 4.1 C) pH= 5.9 D) pH= 7.2

C The second equivalence point is the midpoint of the second steep increase in slope. This corresponds to approximately pH=5.9.

(4-15) What is the molecular formula of a compound with an empirical formula of B2H5 and a molar mass of 53.2 g/mol? A) B2H5 B) B3H7 C) B4H10 D) B6H15

C The simplest approach is to determine the molar mass of the empirical formula. B2H5 has a molar mass of 26.6 g/mol. A molecular formula is always a multiple of the empirical formula; doubling this quantity will result in the molar mass given in the question stem. Therefore, the compound must be B4H10.

(6-1) A reaction is found to stop just before all reactants are converted to products. Which of the following could be true about this reaction? A) the reaction is irreversible, and the forward rate is greater than the reverse rate B) the reaction is irreversible, and the reverse rate is too large for products to form C) the reaction is reversible, and the forward rate is equal to the reverse rate D) the reaction is reversible, and the reverse rate is greater than the forward rate

C This scenario likely describes a situation in which a reaction has reached equilibrium very far to the right (with high product concentration and low reactant concentration). This reaction must be reversible because the reaction did not proceed all the way to the right. Any reaction in equilibrium has equal forward and reverse rates of reaction.

(4-10) Which of the following types of reactions generally have the same number of reactants and products? I. double-displacement reactions II. single-displacement reactions III. combination reactions A) I only B) II only C) I and II only D) II and III only

C Typically, both single-displacement and double-displacement reactions have two reactants that swap either one or two components between the two species. Combination reactions have more reactants than products because the reactants combine together to form the product.

(10-15) How many liters of 2M Ba(OH)2 are needed to titrate a 4L solution of 6M H3PO4? A) 1.33 L B) 12 L C) 18 L D) 56 L

C Use the equivalence point equation: NaVa=NbVb. Ba(OH)2 can dissociate to give two hydroxide ions, so its normality is 2Mx2= 4N. H3PO4 can dissociate to give three hydronium ions, so its normality is 6Mx3= 18N. Plugging into the equation, we get (18N)(4L)= (4N)(Vb). Therefore, Vb is 18L.

(2-15) What is the highest-energy orbital of the elements with valence electrons in the n=3 shell? A) s-orbital B) p-orbital C) d-orbital D) f-orbital

C When n=3, l=0,1,or2. The highest value for l in this case is 2, which corresponds to the d subshell. Although the 3d block appears to be part of the 4th period, it sill has the principal quantum number n=3. In general, the subshells within an energy shell increase in energy as follows: s<p<d<f (although there is no 3f subshell).

(11-6) Which of the following is the correct net ionic reaction for the reaction of copper with silver(I) nitrate? A) Cu + AgNO3 --> Cu(NO3)2 + Ag B) Cu + 2 Ag+ + 2 NO3^- --> Cu2+ + 2 NO3^- + 2 Ag C) 2 Ag+ + 2 NO3^- --> 2 NO3^- + 2 Ag+ D) Cu + 2 Ag+ --> Cu2+ + 2 Ag

D A net ionic equation represents each of the aqueous ions comprising the reactants and products as individual ions, instead of combining them as formula units. Thus, A is not a net ionic reaction. The term net means that the correct answer does not include any spectator ions (ions that do not participate in the reaction). In this reaction, nitrate (NO3^-) remains unchanged. Therefore, B and C are eliminated.

(11-5) The following electronic configurations represent elements in their neutral form. Which element is the strongest oxidizing agent? A) 1s2 2s2 2p6 3s2 3p6 4s2 B) 1s2 2s2 2p6 3s2 3p6 4s2 3d5 C) 1s2 2s2 2p6 3s2 3p6 4s2 3d10 4p1 D) 1s2 2s2 2p6 3s2 3p6 4s2 3d10 4p5

D A strong oxidizing agent will be easily reduced, meaning that it will have a tendency to gain electrons. Atoms usually gain electrons if they are one or two electrons away from filling up their valence shell. A has a full 4s-orbital, meaning that it can only gain an electron if it gains an entire subshell. B has a stable, half-full 3d-orbital, so it is unlikely to pick up electrons unless it can gain five. C has only a single electron in the outer shell, which is more likely lost upon ionization. D would fill up its 4p-orbital by gaining one electron, so it is easily reduced.

(9-2) Which phases of solvent and solute can form a solution? I. solid solvent, gaseous solute II. solid solvent, solid solute III. gaseous solvent, gaseous solute A) I and II only B) I and III only C) II and III only D) I, II, and III

D All three choices can make a solution as long as the two components create a mixture that is of uniform appearance (homogeneous). Hydrogen in platinum is an example of a gas in a solid. Brass and steel are examples of homogenous mixtures of solids. The air we breathe is an example of a homogeneous mixture of gases; wile these are more commonly simply referred to as mixtures, they still fit the criteria of a solution.

(10-13) Which of the following is not a characteristic of an amphoteric species? A) atmospheric species can act as an acid or a base, depending on its environment B) amphoteric species can act as an oxidizing or reducing agent, depending on its environment C) amphoteric species are sometimes amphiprotic D) amphoteric species are always nonpolar

D An amphoteric species is one that can act either as an acid or a base, depending on its environment. Proton transfers are class oxidation-reduction reaction, so A and B are true. C is true because many amphoteric species, such as water and bicarbonate, can either donate or accept a proton. D is false because amphoteric species can be either polar or nonpolar in nature.

(12-14) Which of the following is the best explanation for the fact that a larger mass of electrodes are required for lead-acid batteries, as compared to other batteries, to produce a certain current? A) The lead-acid electrolyte, sulfuric acid, is diprotic and incompletely dissociates in solution B) The energy density of lead-acid electrodes is higher than that of other batteries C) The electrolytes in other batteries less readily dissociate than those of lead-acid batteries D) The energy density of lead-acid electrodes is lower than that of other batteries

D Compared to other cell types, lead-acidic batteries have a characteristically low energy density. While A is a true statement, the incomplete dissociation of sulfuric acid does not fully explain the low energy density of lead-acid batteries. C is likely to be an opposite; The more easily the electrodes dissociate, the easier it is to carry out oxidation-reduction reactions with them.

(12-8) Which of the following can alter the emf of an electrochemical cell? A) the mass of the electrodes B) the length of the wire connecting the half-cells C) the overall size of the battery D) the temperature of the solutions in the half-cells

D E*cell is dependent upon the change in free energy of the system through the equation RTlnKeq=nFE*cell. The temperature, T, appears in this equation; thus, a change in temperature will impact the E*cell.

(8-3) A leak of helium gas through a small hole occurs at a rate of 3.22x10^-5 mol/s. How will the leakage rates of neon and oxygen gases compare to helium at the same temperature and pressure? A) neon will leak faster than helium; oxygen will leak faster than helium B) neon will leak faster than helium; oxygen will leak slower than helium C) neon will leak slower than helium; oxygen will leas faster than helium D) neon will leak slower than helium; oxygen will leak slower than helium

D Graham's law of effusion states that the relative rates of effusion of two gases at the same temperature and pressure are given by the inverse ratio of the square roots of the masses of the gas particles. In other words, a gas with a higher molar mass will lead more slowly than a gas with a lower molar mass. Both neon and oxygen gases will leak at slower rates than helium because they both have more mass than helium.

(4-9) Aluminum metal can be used to remove tarnish from silver when the two solid metals are placed in water, according to the following reaction: 3 AgO + 2 Al --> 3 Ag + Al2O3 This reaction is a: I. double-displacement reaction II. single-displacement reaction III. oxidation-reduction reaction IV. combination reaction A) II only B) IV only C) I and III only D) II and III only

D In the reaction, there is a single displacement, with the silver in silver oxide being replaced by the aluminum to form aluminum oxide. This single-displacement reaction also necessitates a transfer of electrons in an oxidation-reduction reaction; silver, for example, changes from the +2 oxidation state to neutral. Aluminum changes from neutral to the +3 oxidation state.

(5-13) (**insert pic here**) Point C in this reaction profile refers to the: A) reactants B) products C) transition state D) intermediates

D Intermediates exist at "valleys" in reaction diagrams. Reactants (A) are represented by point A. Products (B) are represented by point E. Transition states (C) are represented by points B and D.

(2-14) Why do halogens often form ionic bonds with alkaline earth metals? A) the alkaline earth metals have much higher electron affinities than the halogens B) by sharing electrons equally, the alkaline earth metals and halogens both form full octets C) within the same row, the halogens have much smaller atomic radii than the alkaline earth metals D) the halogens have much higher electron affinities than the alkaline earth metals

D Ionic bonds are formed through unequal sharing of electrons. These bonds typically occur because the electron affinities of the two bonded atoms differ greatly. For example, the halogens have high electron affinities because adding a single electron to their valence shells would create full valence shells. In contrast, the alkaline earth metals have very low electron affinities and are more likely to be electron donors because the loss of two electrons would leave them with full valence shells. A states the opposite and is incorrect because the halogens have high electron affinity and the alkaline earth metals have low electron affinity. B is correct because equal sharing of electrons is a classic description of covalent bonding, not ionic. C is true, but is not relevant to why ionic bonds form.

(4-1) Which of the following best describes ionic compounds? A) ionic compounds are formed from molecules containing two or more atoms B) ionic compounds are formed of charged particles and are measured by molecular weight C) ionic compounds are formed of charged particles that share electrons equally D) ionic compounds are 3-D arrays of charged particles

D Ionic compounds are composed of atoms held together by ionic bonds. Ionic bonds dissociate charged particles with large differences in electronegativity. Rather than forming molecules or being measured by molecular weight (A and B), ionic compounds form large arrays of ions in crystalline solids and are measured with formula weights. In ionic bonds, electrons are not really shared but rather are donated from the less electronegative atom to the more electronegative atom, eliminating C.

(9-14) A saturated solution of cobalt(III) hydroxide (Ksp=1.6x10^-44) is added to a saturated solution of thallium (III) hydroxide (Ksp=6.3x10^-46). What is likely to occur? A) Both cobalt(III) hydroxide and tallium (III) hydroxide remain stable in solution B) Cobalt(III) hydroxide precipitates and thallium(III) hydroxide remains stable in solution C) Thallium(III) hydroxide precipitates and cobalt(III) hydroxide remains stable in solution D) Both thallium(III) hydroxide and cobalt(III) hydroxide precipitate

D Since both salts have a formula MX3 (one of one particle, three of another), it is possible to directly compare the molar solubilities of each. When the solutions are mixed, [OH-] is above saturation levels for both the cobalt and the thallium in the solution. Since thallium(III) hydroxide has a smaller Ksp than that of cobalt(III) hydroxide, it will react first. The ion product of the mixed solution is higher than the Ksp for thallium(III) hydroxide, and the system will shift left to precipitate solid thallium(III) hydroxide. After the thallium(III) hydroxide precipitates, a small excess of OH- will remain, which gives an ion product slightly above the Ksp of cobalt(III) hydroxide. This will cause a small amount of (1-3%) of cobalt(III) hydroxide to also precipitate.

(2-13) Which of the following atoms or ions has the largest effective nuclear charge? A) Cl B) Cl- C) K D) K+

D The effective nuclear charge refers to the strength with which the protons in the nucleus can pull on the electrons. This phenomenon helps to explain electron affinity, electronegativity, and ionization energy. In A, the nonionized chlorine atoms, the nuclear charge is balanced by the surrounding electrons; 17p+/17e-. The chloride ion (B) has a lower effective nuclear charge because there are more electrons than protons; 17p+/18e-. Elemental potassium (C) has the lowest effective nuclear charge because it contains additional inner shells that shield its valence electron from the nucleus. Ionic potassium (D) has a higher effective nuclear charge than any of the other options because it has the same electron configuration as Cl- (and the same amount of shielding from inner shell electrons as neutral Cl) but contains two extra protons in its nucleus: 19p+/18e-.

(9-5) The process of formation of a salt solution can be better understood by breaking the process into three steps: 1) breaking the solute into its individual components 2) making room for the solute in the solvent by overcoming intermolecular forces in the solvent 3) allowing solute-solvent interactions to occur to form the solution Which of the following correctly lists the enthalpy changes for these three steps, respectively? A) endothermic, exothermic, endothermic B) exothermic, endothermic, endothermic C) exothermic, exothermic, endothermic D) endothermic, endothermic, exothermic

D The first step will most likely be endothermic because energy is required to break molecules apart. The second step is also endothermic because the intermolecular forces in the solvent must be overcome to allow incorporation of solute particles. The third step will most likely be exothermic because polar water molecules will interact with the dissolved ions, creating a stable solution and releasing energy.

(8-6) An 8.00g sample of NH4NO3 (s) is placed into an evacuated 10L flask and heated to 227*C. After the NH4NO3 completely decomposes, what is the approximate pressure in the flask? NH4NO3 (s) --> N2O (g) + H2O (g) A) 0.410 atm B) 0.600 atm C) 0.821 atm D) 1.23 atm

D The first thing to do is balance the given chemical equation: NH4NO3 (s) --> N2O (g) + 2 H2O (g). The mass given is 8.00 g, which represents 0.1 mol NH4NO3 (molar mass= 80.0 g/mol). When 0.1 mol of the solid decomposes, it will form 0.1 mol N2O and 0.2 mol water. This gives approximately 0.3 moles of gas product. The ideal gas equation can be used to obtain the pressure in the flask: P= nRT/V= (0.3 mol)(0.0821 L*atm/mol*K)(500K) /(10L)~ 15x0.08= 1.2 atm.

(3-4) Which of the following correctly ranks to compounds below by ascending boiling point? I. Acetone II. KCl III. Kr IV. Isopropyl alcohol A) I<II<IV<III B) III<IV<I<II C) II<IV<I<III D) III<I<IV<II

D The key is to understand the types of intermolecular forces that exist in each of these molecules because larger intermolecular forces correspond to higher boiling points. Kr is a noble gas with a full octet, so the only intermolecular forces present are London dispersion forces, the weakest type of intermolecular force. Acetone and isopropyl alcohol can also form hydrogen bonds, increasing its boiling point. Finally, the strongest interactions are ionic bonds, which exist in potassium chloride.

(1-5) Which of the following atoms only has paired electrons in its ground state? A) sodium B) iron C) cobalt D) helium

D The only answer choice without unpaired electrons in its ground state is helium. Recall that a diamagnetic substance is identified by the lack of unpaired electrons in its shell. A substance without unpaired electrons, like helium, cannot be magnetized by an external magnetic field and is actually slightly repelled. Elements that come at the end of a block (Group IIA, the group containing Zn, and the noble gases, most notably) have only paired electrons.

(1-6) An electron returns from an excited state to its ground state, emitting a photon at lambda= 500nm. What would be the magnitude of the energy change if one mole of these photons were emitted? (note: h=6.626x10^-34 J*s, NA= 6.02x 10^23 mol^-1) A) 3.98 x10^-21 J B) 3.98 x 10^-19 J C) 2.39 x10^3 J D) 2.39 x10^5 J

D The problem requires the MCAT favorite equation E= hc/lambda, where h=6.626x10^-32 J*s (Planck's constant), c=3.00x10^8 m/s is the speed of light, and lambda is the wavelength of the light. this question asks for the energy of one mole of photons, so we must multiply by Avogradro's number, NA= 6.02x10^23 mol^-1.

(5-5) Which of the following experimental methods should never affect the rate of a reaction? A) placing an exothermic reaction in an ice bath B) increasing the pressure of a reactant in a closed container C) putting the reactants into an aqueous solution D) removing the product of an irreversible reaction

D The question asks which alteration does not affect the rate of the reaction. Temperature directly affects the rate constant (k), making A incorrect. Changing the partial pressure of a gas will affect the number of effective collisions per time. This makes B incorrect- but note that concentration changes will not affect the rate of zero-order reactions. Solvents affect the rate of reactions depending on how the reactants interact with the solvent, making C incorrect. Removing the product of an irreversible reaction, D, should not affect the rate of the reaction because the rate law does not depend on the concentrations of products.

(10-6) What is the pH of a solution with an ammonium concentration of 70 mM and an ammonia concentration of 712 mM? (pKb of ammonia is 3.45) A) 2.45 B) 4.45 C) 9.55 D) 11.55

D The question is asking for pH, but because of the information given, we must first find the pOH and then subtract it from 14 to get the pH. pOH=pKb+log[conjugate acid]/[base]= 3.45+log70mM/712mM~ 3.45+log1/10= 3.45-1= 2.45 14-2.45= 11.55=pH

(9-15) The following equilibrium exists when AgBr (Ksp= 5.35x10^-13) is in solution: AgBr (s) <--> Ag+ (aq) + Br- (aq) What is the solubility of AgBr in a solution of 0.0010 M NaBr? A) 5.35x10^-13 g/L B) 1.04x10^-12 g/L C) 5.35x10^-10 g/L D) 1.04x10^-5 g/L

D The solubility of AgBr can be determined using the Ksp value given in the equation. Some amount of AgBr will dissolve; this is the molar solubility x for these conditions. When AgBr dissociates, there will be x amount of silver(I) formed and x amount of bromide- which is added to the 0.0010 M Br- already present from NaBr. AgBr <--> Ag+ + Br- Ksp=[Ag+][Br-] 5.35x10^-13= (x)(0.0010M+x) Given the Ksp of 5.4x10^-13, x will be negligible compared to 0.0010M. The math can be simplified to 5.35x10^-13= x(0.0010) x= 5.35x10^-13/10^-13= 5.35x10^-10 Therefore, x, the molar solubility, is 5.35x10^-10, which looks like C. However, the units of the answer choices are grams per liter, not molarity, and the result must be multiplies by the molar mass (187.8g/mol) 5.35x10^-10g/mol x 187.8g/mol ~ 5x10^-10x200= 1000x10^-8= 1x10^-5 which is close to D.

(12-10) Which of the following expressions correctly describes the relationship between standard electromotive force and standard change in free energy? A) deltaG*= -nF(E*red,anode-E*red,cathode) B) E*cell=(nF/RT)lnKeq C) E*cell= -(RT/nF)lnKeq D) deltaG*= nF(E*red,anode-E*red,cathode)

D There are only two equations involving standard change in free energy in electrochemical cells: deltaG*=-nFE*cell and deltaG*=-RTlnKeq. Substituting E*cell=E*red,cathode-E*red,anode into the first equation and distributing the negative sign gives choice D. A would be the opposite of deltaG*. Setting the two equations equal to each other, we get RTlnKeq=nFE*cell. Solving for E*cell, we get E*cell=(RT/nF)lnKeq, which is the opposite of B. C incorrectly solves the algebra.

(7-2) A reaction has a positive entropy and enthalpy. What can be inferred about the progress of this reaction from this formation? A) the reaction is spontaneous B) the reaction is nonspontaneous C) the reaction is at equilibrium D) there is not enough information to determine whether the reaction is spontaneous or not

D There is not enough information to determine whether or not the reaction is spontaneous. If the signs on enthalpy and entropy are the same, the reaction is temperature dependent. Without the temperature, we cannot determine if this reaction is spontaneous, nonspontaneous, or at equilibrium.

(12-5) If the value of E*cell is known, what other data is needed to calculate deltaG*? A) equilibrium constant B) reaction quotient C) temperature of the system D) half-reactions of the cells

D This answer comes directly from the equation relating Gibbs free energy and E*cell. deltaG*= -nFE*cell, where n is the number of moles of electrons transferred and F is the Faraday constant, 96485C/mol e-. To determine n, one must look at the balanced half-reactions occurring in the oxidation-reduction reaction.

(10-11) What is the [H3O+] of a 2M aqueous solution of a weak acid HXO2 with Ka=3.2x1-^-5? A) 6.4x10^-5M B) 1.3x10^-4M C) 4.0x10^-3M D) 8.0x10^-3M

D This question requires the application of the acid dissociation constant. weak acids do not dissociate completely; therefore, all three species that appear in the balanced equation will be present in solution. Hydrogen ions and conjugate base anions dissociate in equal amounts, so [H+]=[XO2^-]. If the initial concentration of HXO2 was 2M and some amount of x dissociates, we will have x amounts of H3O+ and XO2^- at equilibrium, with 2 M-x amount of HXO2 at equilibrium. Ka=[H3O+][XO2^-]/[HXO2]= (x)(x)/ 2 M-x ~ x^2/2 x^2/2= 3.2x10^-5 --> x^2= 6.4x10^-5= 64x10^-6--> x=8x10^-3M

(5-14) The following system obeys second-order kinetics: 2 NO2 --> NO3 + NO (slow) NO3 + CO --> NO2 + CO2 (fast) What is the rate law for this reaction? A) rate=k[NO2][CO] B) rate= k[NO2]2[CO] C) rate= k[NO2][NO3] D) rate= k[NO2]2

D To answer this question, recall that the slow step of a reaction is the rate-determining step. The rate is always related to the concentrations of the reactants in the rate-determining step (not the overall reaction), so NO2 is the only compound that should be included in the correct answer. The concentration of NO2 is squared in the rate law because the question tell us that the system obeys second-order kinetics.

(11-8) During the assigning of oxidation numbers, which of the following elements would most likely be determined last? A) Ar B) F C) Sr D) Ir

D When assigning oxidation numbers, one starts with elements of known oxidation state first, and determines the oxidation state of the other elements by deduction. As a Noble gas, argon will always have an oxidation state of zero. As a Group VIIA element, fluorine will have an oxidation state of 0 by itself or -1 in a compound. As a Group IIA element, strontium will have an oxidation state of 0 by itself or +2 in a compound. Like most transition metals, Iridium can have various oxidation states ranging from -3 to +8. Therefore, one would have to determine the oxidation states of other atoms in an iridium-containing compound to determine iridium's oxidation number.

(7-3) Pure sodium metal spontaneously combusts upon contact with room temperature water. What is true about the equilibrium constant of this combustion reaction at 25 degrees Celsius? A) Keq<0 B) 0<Keq<1 C) Keq=1 D) Keq>1

D deltaG*rxn= -RTlnKeq deltaG*rxn is negative (as it must be for a spontaneous reaction), and R and T are always positive. Therefore, lnKeq must also be positive for the sign convention to work out correctly. Since ln(1)=0, the natural log of any number greater than 1 will be positive, and the natural log of any number less than 1 will be negative. In order for lnKeq to be a positive number, Keq must be greater than 1.

(5-15) The potential energy diagram shown represents four different reactions: (**insert pic here**) Assuming identical conditions, which of the reactions displayed on the energy diagram proceeds the fastest? A) A B) B C) C D) D

D The faster a reaction can reach its activation energy, the faster it will proceed to completion. Because this question states that all conditions are equal, the reaction with the lowest activation energy will have the fastest rate. In the diagram, D has the lowest activation energy.


Conjuntos de estudio relacionados

FB Digital Marketing Associate Quiz

View Set

Chapter 7 Translation of foreign Currency Financial Statements.

View Set

TOK: Week of 12/4: Common Logical Fallacies quiz

View Set

Legal Environment of Business Final

View Set

ATI Achieve: Preparing for Clinical Experiences

View Set

proving lines are parallel and using slope

View Set